Vous êtes sur la page 1sur 88

What is an ideal management control system

Management control is a process of assuming that resources are obtained and used effectively and
efficiently in the accomplishment of the organization’s objectives. It is a fundamental necessity for the success of
a business and hence from time to time the current performance of the various operations is compared to a
predetermined standard or ideal performance and in case of variance remedial measures are adopted to confirm
operations to set plan or policy.

Features of management control system

 Total System: MANAGEMENT CONTROL SYSTEM is an overall process of the enterprise which aims to
fit together the separate plans for various segments as to assure that each harmonizes with the others and that
the aggregate effect of all of them on the whole enterprise is satisfactory.

 Monetary Standard: MANAGEMENT CONTROL SYSTEM is built around a financial structure and all
the resources and outputs are expressed in terms of money. The results of each responsibility centre in
respect to production and resources are expressed in terms of a common denominator of money.

 Definite pattern: It follows a definite pattern and time table. The whole operational activity is regular and
rhythmic. It is a continuous process even if the plans are changed in the light of experience or technology.

 Coordinated System: It is a fully coordinated and integrated system.

 Emphasis: Management control requires emphasis both on the search for planning as well as control. Both
should go hand in hand to achieve the best results.

 Function of every manager: Manager at every level as to focus towards future operational and accounting
data, taking into consideration past performance, present trends and anticipated economic and technological
changes. The nature, scope and level of control will be governed by the level of manager exercising it.

 Existence of goals and plans: MANAGEMENT CONTROL SYSTEM is not possible without
predetermined goals and plans. These two provide a link between such future anticipations and actual
performance.

 Forward looking: MANAGEMENT CONTROL SYSTEM is on the basis of evaluation of past performance
that the future plans or guidelines can be laid down. Management Control involves managing the overall
activity of the enterprise for the future. It prevents deviations in operational goals.

 Continuous process: It is a continuous process over the human and material resources. It demands vigilance
at every step. Deciding, planning and regulating the activities of people associated in the common task of
attaining the objectives of the organization is a the primary aim of MANAGEMENT CONTROL SYSTEM.

 People oriented: It is the managers, engineers and operators which implement the ideas and objectives of
the management. The coordination of the main division of an organization helps in smoother operations and
less friction which results in the achievement of the predetermined objectives.

Scope of control
MANAGEMENT CONTROL SYSTEM is an important process in which accounting information is used to
accomplish the organizations objectives. Therefore the scope of control is very wide which covers a very wide
range of management activities.
 Policies control: Success if a business depends on formulation of sound policies and their proper
implementation.

 Control over organization: It involves designing and organizing the various departments for the smooth
running of the business. It attempts to remove the causes of such friction and rationalizes the organizational
structure as and when the need arises.

 Control over personnel: Anything that the business accomplishes is the result of the action of those people
who work in the organization. It is the people, and not the figures, that get things done.

1
 Control over costs: The cost accountant is responsible to control cost sets, cost standards, labour material
and over heads. He makes comparisons of actual cost data with standard cost. Cost control is a delicate task
and is supplemented by budgetary control systems.

 Control over techniques: It involves the use of best methods and techniques so as to eliminate all wastages
in time, energy and material. The task is accomplished by periodic analysis and checking of activities of
each department with a view to avoid an eliminate all non-essential motions, functions and methods.

 Control over capital Expenditure: Capital budget is prepared for the whole concern. Every project is
evaluated in terms if the advantage it accrues to the firm. For this purpose capital budgeting, project
analysis, study of cost of capital etc are carried out.

 Overall control: A master plan is prepared for overall control and all the departments of the concern are
involved in this procedure.

What is the concept of free cash flow as applied to organization? Explain the process of computation.

We define net cash flow as net income plus non cash adjustment which typically means net income plus
depreciation though that cash flows cannot be maintained over time unless depreciated fixed assets are replaced.
So management is not completely free to use its cash flows however it chooses. Therefore we define the term
free cash flows.

Free cash flow is the cash flow actually available for distribution to investor after the company has made all the
investment in fixed assets and working capital necessary to sustain ongoing operation. When we studied income
statement in accounting the emphasis was probably on the firm’s net income, which is accounting profit.
However the value of company’s operation is determined by the stream of cash flows that the operations will
generate now and in the future. To be more specific, the value of operation depends on all the future expected
free cash flows, defined as after- tax operating profit minus the amount of new investment in working capital and
fixed assets necessary to sustain the business. Therefore the way for managers to make their companies more
valuable is to increase their free cash flow.

Uses of FCF:

1. Pay interest to debt holders, keeping in mind that the net cost to the company is the after tax interest
expense.
2. Repay debt holders, that is, pay off some of debt.
3. Pay dividends to shareholders.
4. Repurchase stock from shareholders.
5. Buy marketable securities or other non operating assets.
In practice, most companies combine these five uses in such a way that the net total is equal to FCF. For
example, a company might pay interest and dividends, issue new debts, also sell some of its marketable
securities. Some of these activities are cash outflows (paying interest and dividends) and some are cash inflows
(issuing debt and selling marketable securities), but the net cash flow from these five activities is equal to free
cash flows.

Computation of free cash flows:


Eg:
Suppose the company had a 2001 NOPAT of $170.3million and depreciation is only the non cash charge which
is $100million then its operating cash flow in 2001 would be NOPAT plus any non cash adjustment on the
statement of cash flows.
2
Operating cash flow =NOPAT +depreciation (non cash adjustment)
= $17.03 + $100
= $270.3

Company has $1,455million operating assets, at the end of 2000, but $1,800 at the end of 2001.it made a net
investment in operating assets of

Net investment in operating assets = $18, 00 - $1,455 = $345million

If net fixed assets rose from $870million to $1000million however company reported $100million of
depreciation. So its gross investment in fixed assets would be

Gross investment = net investment + depreciation


= $130 + $100 = $230million

Company free cash flows in 2001 was

FCF = operating cash flow – gross investment in operating assets


= $270.3 - $445
= - $174.7million

An algebraically equivalent equation is

FCF = NOPAT - Net investment in operating assets


= $170.3- $345
= - $174.7million

Even though company had a positive NOPAT, its very high investment in operating assets resulted in a
negative free cash flow. Because free cash flow is what is available for distribution to investor, not only was
there nothing for investors, but investor actually had to provide additional money to keep the business ongoing.
A negative current FCF not necessarily bad provided it is due to the high growth or to support the growth. There
is nothing wrong with profitable growth; even it causes negative free cash flow in the short term

What is Balance Scorecard? What is the process of implementation and difficulties in implementation?

The Balanced Scorecard (BSC) is a performance management tool which began as a concept for
measuring whether the smaller-scale operational activities of a company are aligned with its larger-scale
objectives in terms of vision and strategy.

By focusing not only on financial outcomes but also on the operational, marketing and developmental
inputs to these, the Balanced Scorecard helps provide a more comprehensive view of a business, which in turn
helps organizations act in their best long-term interests.

Organizations were encouraged to measure—in addition to financial outputs—what influenced such


financial outputs. For example, process performance, market share / penetration, long term learning and skills
development, and so on.

The underlying rationale is that organizations cannot directly influence financial outcomes, as these are
"lag" measures, and that the use of financial measures alone to inform the strategic control of the firm is unwise.
Organizations should instead also measure those areas where direct management intervention is possible. In so
doing, the early versions of the Balanced Scorecard helped organizations achieve a degree of "balance" in
selection of performance measures. In practice, early Scorecards achieved this balance by encouraging managers
to select measures from three additional categories or perspectives: "Customer," "Internal Business Processes"
and "Learning and Growth."

The balance scorecard suggests that we view the organization from four perspectives, and to develop
metrics, collect data and analyze it relative to each of these perspectives:

3
• The learning and growth perspective : “To achieve our vision, how will we sustain our ability to
change and improve?”

• The business process perspective : “To satisfy our shareholders and customers what business
processes must we excel at?”

• The customer perspective : “To achieve our vision, how should we appear to our customer?”

• The financial perspective : “To succeed financially, how should we appear to our shareholders?”

Implementing a
Balanced
Scorecard

We can summarize the


implantation of a balanced scorecard in four general steps;
1. Define strategy.
2. Define measure of strategy.
3. Integrate measures into the management system.
4. Review measures and result frequently.

Each of these steps is iterative, requiring the participation of senior executive and employees throughout the
organization

Define Strategy
The balance scorecard builds a link between strategy and operational action. As a result it is necessary to begin
the process of defining a balanced scorecard by defining the organization goals are explicit and what that targets
have been developed.

Define Measures of Strategy


The next step is to develop measures in support of the articulate strategy. It is imperative that the organization
focuses on a few critical measures at this point; otherwise management will be overloaded with measures. Also, it
is important that the individual measures be linked with each other in a cause effect manner

Integrated Measures into the management system


The balanced scorecard must be integrated with the organization formal and informal structure, its culture, and its
human resources practice. While the balanced Scorecard gives some means for balancing measures, the measures
can still become unbalanced by others system in the organization such as compensation policies that compensate
the manager strictly based on financial performance.

Review Measures and result Frequently


4
Once the balance scorecard is up and running it must be consistently reviewed by senior management. The
organization should be looking for the following

• How do the outcome measures say the organization is doing?


• How do the driver measures say the organization is doing?
• How has the organization’s strategy changed since the last review?
• How has the scorecard measures changed?

The most important aspects of these reviews are as follows;

• They tell management whether the strategy is being implemented correctly and how
successfully the strategy is working.
• They show that management is serious about the importance of these measures.
• They maintain alignment of measure to ever changing strategies.

Difficulties in implementing Balanced Scorecard

The following problems unless suitably dealt with, could limit the usefulness of the balanced scorecard approach:
• Poor correlation between nonfinancial measures and result.
• Fixation on financial result. No mechanism for improvement.
• No mechanism for improvement.
• Measures overload.

Poor Correlation between Nonfinancial measures and result


Simply put there is no guarantee that future profitably will allow targets achievement in any nonfinancial area.
This is probably the biggest problem with the balanced scorecard because there is an inherent assumption that
future profitability does follow from achieving the scorecard measures, identifying the cause effect relationships
among the different measures is easier said than done.

This will be a problem with any system that is trying to develop proxy measures for future performance. While
this does not mean that the balanced Scorecard should be abandoned it is imp that comp adopting such a system
understand that the links between nonfinancial measures and financial performance are still poorly understood.

Fixation on Financial Results


As previously discussed not only are most senior managers well trained and very adept with financial measures
but they also most keenly feel pressure regarding the financial performance of their comp. Shareholder are vocal
and the board of directors often applies pressure on the stakeholders behalf .this pressure often overwhelms the
long term uncertain payback of the nonfinancial measures.

Non mechanism for Improvement


One of the most overlooked pitfalls of the balanced scorecard is that a company cannot achieve Stretch goals if
the Company has no mechanism for improvement .Unfortunately achieving many of these goals require complete
shifts in the way that business is done yet the company often does not have mechanism to make those shifts . The
mechanism available takes additional resource and requires a changed in the company culture. These changes do
not happen overnight nor do they respond automatically to a new stretch targets. Inertia often works against the
company employees are accustomed to a self limited cycle of setting targets, missing those targets and readjusting
the targets to reflect what was actually achieved. Without a method for making improvement, improvements are
unlikely to consistently happen no matter how good the stretch goal sound.

Measurement overload
How many critical measures can one manager track at one time without losing? Unfortunately there is no right
answer to this question except it is more than 1 and less than 50. It too few then the manager is ignoring measures
that are critical to creating success. If it too many then the manager may risk losing focus and trying to do too
many things at once.

5
Girish Engineering (MCS-2004) Numerical

Responsibility budgeting was introduced in a medium sized organization Girish Engineering.

Monthly report (in part) for an expense centre in factory is:


[All figures in Rs. Lacs]

Actual Variance
Direct Labour 100.13 0.21 (Favourable)
Indirect Labour 66.34 8.10 (Unfavourable)
Total Controllable Costs 168.47 8.50 (Unfavourable)
Department Fixed Costs 38.82 --------
Allocated Costs 53.62 --------

Questions:
a. Why no variance is shown in two items? Is this correct approach in performance reporting?
b. Should overhead expenses mentioned above be included in Controllable Costs? Why? Why not?

Solution (a):

Variances between actual and budgeted departmental fixed costs are obtained simply by subtraction, since these
costs are not affected by either the volume of sales or the volume of production. That’s why no variance is
shown for departmental fixed costs.

Allocated costs are a share of the costs of a resource used by a project, where the same resource is also used by
other activities. These are different to the Incurred costs because these costs are not exclusively related to any
individual project. However, the cost of the resource still needs to be recovered, and making a fair and
reasonable charge to all projects using the resource does this.

The key difference between costs and Allocated costs is that the latter will be charged based upon an estimate,
rather than actual cash values. Thus as it is charged based upon an estimate the budgeted figure is the same as the
actual figure and hence no variances.

Solution (b):
Overhead Expenses mentioned above should not be included in controllable costs because some costs are
uncontrollable like fixed costs. . They don't vary with the change in short run managerial decisions and output.
And some costs are controllable i.e. they can be managed and changed with the managerial decisions and output.

As the above overhead expenses would have certain portion of fixed expenses this is hard to control. So, these
should not be a part of controllable cost.

ABC ltd. (MCS-2008) Numerical

Particulars Division X (Rs.) Division Y (Rs.)


ROI 28% 26%
Sales 100 Lacs 500 lacs
Investment 25 lacs 100 Lacs
EBIT 7 Lacs 26 lacs

Analyze and comment upon performances of both the divisions

Solution:

6
Division X
ROI = (Profit / investment)* 100
Profit = (28/100)*25lacs
= 7lacs

Profit margin = (Profit/sales)*100


= (7/100)*100
= 7lacs

Turnover of investments = (Sales/investment)*100


= (100/25)*100
= 4 times

Division Y
ROI = (Profit / investment)* 100
Profit = (26/100)*100lacs
= 26lacs

Profit margin = (Profit/sales)*100


= (26/500)*100
= 5.2lacs

Turnover of investments = (Sales/investment)*100


= (500/100)*100
= 5 times

Profit margin of X is better than profit margin of division Y. Turnover of investment of division Y is better than
Division X.

Hence cost management of Division X is better than Division Y.

MCS designers apparently disagree whether single measure to evaluate the profit performance and
capital investment performance is preferable or SEPARATE measures for each are preferable –
COMMENT
There should be different measures used for evaluating profit performance and capital investment performance
as needed.
The goal of performance measurement systems is to implement strategy. In setting up such systems, senior
management selects measures that best represent the company's strategy. These measures can be seen as current
and future critical success factors; if they are improved, the company has implemented its strategy. The strategy's
success depends on its soundness. A performance measurement system is simply a mechanism that improves the
likelihood the organisation will implement its strategy successfully.
Measuring Profitability
There are two types of profitability measurements used in evaluating a profit center, just as there are in
evaluating an organization as a whole. First, there is a measure of management performance, which focuses on
how well the manager is doing. This measure is used for planning, coordinating, and controlling the profit
center's day-to-day activities and as a device for providing the proper motivation for its manager. Second, there
is the measure of economic performance, which focuses on how well the profit center is doing as an economic
entity. The messages conveyed by these two measures may be quite different from each other. For example, the
management performance report for a branch store may show that the store's manager is doing an excellent job
under the circumstances, while the economic performance report may indicate that because of economic and
competitive conditions in its area the store is a losing proposition and should be closed.
The necessary information for both purposes usually cannot be obtained from a single set of data. Because the
management report is used frequently, while the economic report is prepared only on those occasions when
economic decisions must be made, considerations relating to management performance measurement have first
priority in systems design-that is, the system should be designed to measure management performance routinely,
with economic information being derived from these performance reports as well as from other sources.
7
Capital Investment Measurement
Most proposals require significant new capital. Techniques for analyzing capital investment proposals attempt to
find either
(a) The net present value of the project, that is, the excess of the present value of the estimated cash inflows over
the amount of investment required, or
(b) The internal rate of return implicit in the relationship between inflows and outflows. An important point is
that these techniques are used in only about half the situations in which, conceptually, they are applicable.
There are at least four reasons for not using present value techniques in analyzing all proposals.
1. The proposal may be so obviously attractive that a calculation of its net present value is unnecessary. A
newly developed machine that reduces costs so substantially that it will pay for itself in a year is an
example.

2. The estimates involved in the proposal are so uncertain that making present value calculations is believed to
be not worth the effort-one can't draw a reliable conclusion from unreliable data. This situation is common
when the results are heavily dependent on estimates of sales volume of new products for which no good
market data exist. In these situations, the "payback period" criterion is used frequently.
3.
4. The rationale for the proposal is something other than increased profitability. The present value approach
assumes that the "objective function" is to increase profits, but many proposed investments win approval on
the grounds that they improve employee morale, the company's image, or safety.
5.
6. There is no feasible alternative to adoption. Environmental laws may require investment in a new program,
as an example.
The management control system should provide an orderly way of deciding on proposals that cannot be
analyzed by quantitative techniques. Systems that attempt to rank non-quantifiable projects in order of
profitability won't work. Many projects do not fit into a mechanical ranking scheme.
What are the different methods to measure profits of a profit center in organizations? Which different

messages each type of measure is likely to convey to managers?

When financial performance in a responsibility center is measured in terms of profit, which is the difference
between the revenues and expenses, the responsibility center is called a profit center.

Profit as a measure of performance is especially useful since it enables senior management to use one
comprehensive measure instead of several measures that often point to different directions.

There are two types of profitability measurements in a profit center, just as there are for the organization as a
whole. There is, first, a measure of management performance, in which the focus is on how well the manager
is doing. This measure is used for planning, coordinating and controlling the day-to-day activities of the profit
center. Second, there is a measure of economic performance, in which the focus is on how well the profit center
is doing as an economic entity. The message given by these two measures may be quite different.

Types of Profitability measures:

In order to evaluate the economic performance of a profit center, one must use net income after allocating all
costs. However, in evaluating the performance of manager, any of five different measures of profitability can be
used.

1) Contribution Margin: The logic behind using contribution margin as a measure is that fixed expenses are
not controllable by the manager, and therefore he should focus on maximizing the spread between revenue
and expenses. But the problem with this is that some fixed costs are controllable and all fixed costs are
partially controllable. A focus on the contribution margin tends to direct attention away from this
responsibility.
8
2) Direct Profit: This measure shows the amount that the profit center contributes to the general overhead and
profit of the corporation. It incorporates all expenses incurred in or directly traced to the profit center,
regardless of whether these items are entirely controllable by the profit center manager. A weakness of this
measure is that it does not recognize the motivational benefit of charging headquarters costs.

3) Controllable Profit: Headquarters expenses are divided into two categories: controllable and non-
controllable. The controllable expenses are controlled by business unit manager. Consequently, if these costs
are included in the management system, the profit will be after the deduction of all expenses that are
influenced by profit center manager.

4) Income before Taxes: In this measure, all corporate overhead is allocated to profit centers. The basis of
allocation reflects the relative amount of expense that is incurred for each profit center. If corporate
overheads are allocated to profit centers, budgeted costs, not actual costs, should be allocated. Then the
performance report will show an identical amount in the “budget” and “actual” columns for such overheads.

5) Net Income: Here, companies measure performance of domestic profit centers at the bottom line, the
amount of net income after income tax. There are two arguments 1) Income after tax is constant percentage
of the pretax income, so there is no advantage in incorporating income taxes 2) many decisions that have
impact on income taxes are made at headquarters, and it is believed that profit center manager should not be
judged by the consequences of these decisions.

Explain special characteristics of professional organizations which impact Management Control. What

are interactive controls?

1. Goals
A dominant goal of a manufacturing company is to earn a satisfactory profit, specifically a satisfactory return on
assets employed. A professional organization has relatively few tangible assets; its principal asset is the skill of
its professional staff, which 'doesn't appear on its balance sheet. Return on assets employed, therefore, is
essentially meaningless in such organizations. Their financial goal is to provide adequate compensation to the
professionals.

In many organizations, a related goal is to increase their size. In part, this reflects the natural tendency to
associate success with large size. In part, it reflects economies of scale in using the efforts of a central personnel
staff and units responsible for keeping the organization up-to-date. Large public accounting firms need to have
enough local offices to enable them to audit clients who have facilities located throughout the world.

2. Professionals
Professional organization is labour intensive and the labour is of a special type. Research and development
organization use in setting selling price and for other management purposes .standard cost system ,separation of
fixed and variable cost and analyses of variance were built on the foundation are example of organization whose
product are professional service. Professional tends to give in adequate weight to the financial implication of their
decision they want to do the best job they can regardless of its cost.
Because profession are the organization most important resource some authors have advocated that the value of
these profession should be counted as assets the system that does this is called human resource accounting .in the
1970’s many books and articles were written on this subject but few comp actually such a system and we do not
know of any that one current .the problem of measuring the value of human assets is intractable.

3. Output and input measurement

9
The output of a profession organisation cannot be measured in physical terms, use in setting selling price and for
other management purposes .standard cost system, seperstion of fixed and variable cost and analyses of variance
were built on the foundation. We can measures the number of patient a physician treats n a day and even classify
these visit by type of complaint but this is by no means equivalent to measuring the amt or quality earned is one
measures of output in some professional organization but these monetary amts at most relate to the quantity of
service rendered not to their quality.
Some profession notably scientist engineer, and professional are reluctant to keep track of how they spend their
time and this complicate the track of measuring performance .this reluctant seems to have its root in tradition
usually it can be overcome if senior management is willing to put appropriate emphasis on the necessity for
accurate time reporting .nevertheless difficult problem arise in deciding how time should be charged to clients .if
the normal work week is 40 hrs should a job be charged for 1/40th of a week compensation for each other spent on
it? If so how should work done on evening and weekend be counted how to account for time spent reading
literature ,going to meeting ,and otherwise keeping up to date?

4. Small Size
With a few exception such as some law firm and accounting firms ,professional organisations are relatively small
and operate at a single location .senior management in such organisations can personally observe what is going on
and personally motivate employee .thus there is less need for a sophisticated management control system ,with
profit centres and formal performance reports nevertheless even a small organisations need a budget a regular
comparison of performance against budget ,and a way relating compensation to performance.

5. Marketing
In a manufacturing company there is a dividing line between marketing activities and production activities only
senior management is concerned with both .such a clean separation does not exist in most Professional
organisation, however their time and this complicate the track of measuring performance .this reluctant seems to
have its root in tradition usually it can be overcome if senior management is willing to put appropriate emphasis
on the necessity for accurate time reporting. Nevertheless difficult problem arise in deciding how time should be
charged to clients .if the normal work. These marketing activities are conducted by professional usually by
professional, usually by professional who spend much of their time in production work that is working for clients.
In such situation it is difficult to assign appropriate credit to the person responsible for selling a new customer; in
a consulting firm for example a new engagement may result from a conversation between a member of the firm
or from the reputation of one of the firm professional as an outgrowth of speeches or articles. Moreover the
profession al who is responsible for obtaining the engagement may not personally involved in carrying it out
.until fairly recently these marketing contribution were rewarded subjectively –that is they were taken into
account in promotion and compensation decisions .some organisation now give explicit credit, perhaps as a
percentage of the project revenue, if the person revenue, if the person who hold sold the project can be identified.

What is Interactive Control?


Interactive control alerts management of strategic uncertainties either trouble or opportunities that become the
basis for manager to adapt to a rapidly changing environments by thinking about new strategies.
1. A subset of the management control information that has a bearing on the strategic uncertainties
facing the buss becomes the focal point.
2. Senior executive take such information seriously.
3. Managers at all levels of the org focus attention on the information produced by the system.

How do we evaluate the Performance Appraisal?


As noted earlier in regard to teachers, at the extremes the performance of professionals is easy to judge.
Appraisal of the large percentage of professionals who are within the extremes is much more difficult. For some
professions, objective measures of performance are sometimes unavailable: The recommendations of an
investment analyst can be compared with actual market behavior of the securities; the accuracy of a surgeon's
diagnosis can be verified by an examination of the tissue that was removed; and the doctors' skill can be
measured by the success ratio of operations. These measures are, of course, subject to appropriate qualifications,
and in most circumstances the assessment of performance is finally a matter of human judgment by superiors,
peers, self, subordinates, and clients. Judgments made by superiors are the most common. For these, professional

10
organizations increasingly use formal systems to collect performance appraisals as a basis for personnel
decisions and for discussion with the professional. Some systems require numerical ratings of specified attributes
of performance and provide for a weighted average of these ratings. Compensation may be tied, in part, to these
numerical ratings. In a matrix organization, both the project leader and the head of the functional unit that is the
professional's organizational "home" judge performance. "

Appraisals by a professional's peers, or by subordinates, are sometimes part of a formal control system. In some
organizations, individuals may be asked to make a self-appraisal. Expressions of satisfaction or dissatisfaction
from clients are also an important basis for judging performance, although such expressions may not always be
readily forthcoming.

The budget can be used as the basis for measuring cost performance, and the actual time taken can be compared
with the planned time. Budgeting and control of discretionary expenses are as important in a professional firm
as in a manufacturing company.

Such financial measures are relatively unimportant in assessing a professional's contribution to the firm's,
profitability, however. The professional's major contribution is related to quantity and above all quality of work,
and its appraisal must be largely subjective. Furthermore, the appraisal must be made currently; it cannot wait
until one learns whether a new building is well designed, a new control system actually works well, or a bond
indenture has a flaw.

In some professions, internal audit procedures are used to control quality. In many accounting firms, the report
of an audit is reviewed by a partner other than the one who is responsible for it, and the work of the whole firm is
"peer reviewed" by another firm. The proposed design of a building may be reviewed by architects who are not
actively involved in the project.

Kiran Company (MCS-2004) Numerical

Budget versus Actual comparison for div Z of Kiran company is as follows:

Budget Actual Actual better (worse)


than budget

Sales and other income 800 740 (60)

Variable expenses 480 436 44

Fixed expenses 120 120 0

Sales promotional expenses 40 28 12

Operating profit 160 156 4

Net working capital 400 412 12

Fixed assets 160 148 (12)

(a) Carry out and overall performance analysis to decide areas needing investigation.

From the given data, we see that there is a certain amount of variance between the budgeted operating profit and
actual operating profit. In order to analyze the variances, we need to understand the key causal factors that affect
profit, namely, revenues and cost structure. The profit budget has embedded in it certain expectations about the
state of total industry, company’s market share, selling prices and cost structure. Results from variance
computation are actionable if changes in actual results are analyzed against each of this expectation.

Revenue variances, that is a negative Rs 60 lakhs, could be a result of selling price variance, mixed variance
and/or volume variance. A combination of above three factors must have been unfavorable that is either the
volume of sales must have been below the budgeted volumes ( this must be particularly true since actual variable
11
expenses are less than budgeted) and/or the selling price must have been below expectation and/or the proportion
of products sold with a higher contribution must have been less than budgeted.

One more factor could have been the overall industry volume. However, this factor is beyond the managements
control and largely dependent on the state of economy.

Variable expenses are directly proportional to volumes and hence as is evident are less than budgeted.

Sales promotional expenses also show a negative variance which could be a cause of lower sales volumes.

A cause of concern is that despite lower sales, the net working capital is more than budgeted which indicates
capital block in higher inventory.

Another issue is that the fixed assets are lower than the budget by Rs 12 lakhs which may indicate slower
capacity expansion then expected or distressed sale of assets to tide over cash flow.

(b) What are the remedial measures if any would you suggest based on analysis?

The budgeted estimates may be too optimistic and far from reality, one needs to ensure that estimates the as
realistic as possible. Given the estimates are correct, in that case depending upon the above analysis, the
management needs to take corrective action areas needing improvement, sales volume could be improved by
better marketing, quality standards and promotional efforts, product mix could be improved by selling more of
higher contribution products. Better sales will ensure a higher inventory turnover. Better credit management to
recover receivables, will ensure improve cash flow situation since less capital will be tied up in working capital.

12
Shandilya Ltd. (MCS-2008) Numerical

Shandilya Ltd. has adopted Economic Value Added (EVA) technique for the appraisal of performance of its
three divisions A,B and C. Company charges 6% for current assets and 8 % for Fixed Assets, while computing
EVA relevant data are given below :-

Particulars Div A Div B Div C Total

Budgete Actual Budgete Actual Budgeted Actual Budgete Actual


d d d

Profit 360 320 220 240 200 200 780 760

Current Assets 400 360 800 760 1200 1400 2400 2520

Fixed Assets 1600 1600 1600 1800 2000 2200 5200 5600

Solution:

Particulars Div A Div B Div C Total

Budgete Actual Budgete Actual Budgeted Actua Budgete Actual


d d l d

ROA 18% 16% 9% 9% 6% 6% 10% 9%

EVA 208 170.4 44 50.4 -32 -60 220 160.8

b) Comment upon both methods, based on results.

There are three apparent benefits of an ROA measure. First, it is a comprehensive measure in that anything that
effects the financial statements is reflected in this ratio. Secondly, ROA is easy to calculate, easy to understand,
and meaningful in absolute sense. Finally, it is a common denominator that may be applied to any organizational
units responsible for profitability, no matter what its size or what business it practices. The performance of
different units may be compared directly to each other. Also, ROI data is available for competitors that can be
used as a basis for comparison. Nevertheless, the EVA approach has some inherent advantages over ROA.

There are three compelling reasons to use EVA over ROI. First, with EVA all business units have the same
profit objective for comparable investments. The ROI approach, on the other hand, provides different incentives
for investment across business units. For example, a business unit that is currently achieving 30% ROA would be
most reluctant to expand unless it is able to earn a ROI of 30% or more on additional assets. Second, decision
that increase a centre’s ROI may decrease its overall profits. Third advantage of EVA is that different interest
rates may be used for different types of assets. For example, a relatively low rate May be used for inventories
while a higher rate may be used for different types of fixed assets.

Describe differences in budgeting perspective of engineered and discretionary expense


centre

Expense centers:

13
Expenses center are responsibility centers for which input or expenses are measured in monetary
terms, but for which outputs are not measured in monetary terms. There are two general types :
engineered expense center and discretionary expense center. They correspond to two types of costs :
Engineered costs are elements of cost for which the right or proper amount of costs that should be
incurred can be estimated with a reasonable degree of reliability. Costs incurred in factory for direct
labour direct material component supplies and utilities are examples.

Engineered expense centers:


Engineered expense center have the following characteristics:
1. Their inputs can be measured in monetary terms.
2. Their output can be measured in physical terms.
3. The optimal dollar amount of input required to produce one unit of output can be established

Engineered expense center usually are found in manufacturing operations. Warehousing, distribution,
trucking and similar units in the marketing organization also may be engineered expense center and so
many certain responsibility center within administrative and support department. Examples are
accounts receivable account payable and payroll section in the controller department personnel record
and cafeteria in the human resource department shareholder record in the corporate secretary
department and the company motor pool. Such units perform repetitive task for which standard cost
can be developed

In an engineered expense center the output multiplied by the standard cost or each unit produced
represents what the finished product should have cost. When this cost is compared to actual costs, the
difference between the two represents the efficiency of the organization unit being measured.

We emphasize that engineered expense centers have other important tasks not measured by cast alone.
The effectiveness of these aspects of performance should be controlled. For example expenses center
supervisor are responsible for the quality of good and for the volume of production in addition to their
responsibility for cost efficiency. Therefore the type and amount of production is prescribed and
specific quality standards are set so that manufacturing costs are not minimized at the expense of
quality. Moreover manager of engineered expense center may be responsible for activities such a
training that are not related to current production judgment about their performance should include an
appraisal of how well they carry out these responsibilities.

There are few if any responsibility center in which all cost items are engineered. Even in highly
automated production department the amount of indirect labour and of various services used can vary
with management discretion.

Thus, the term engineered costs center refers to responsibility center in which engineered cost
predominate but it does not imply that valid engineering estimates can be made for each and every cost
item.

Discretionary expense center:

The output of discretionary expenses center cannot be measured in monetary terms. They include
administration and support units research and development organization and most marketing activities.

The term discretionary does not mean that management judgments are capricious or haphazard.
Management has decided on certain policies that should govern the operation of the company. One
company may have a small headquarter staff another company of similar size and in the same industry

14
may have a staff that is 10 time as large the management of both companies may be concerned that
they made the correct decision on staff size but there is no objective way judging which decision was
actually better manager are hired and paid to make such decision after such a drastic change the level
of discretionary expenses generally has a similar pattern from one year to the next.

The difference between budgeted and actual expense is not a measure of efficiency in a discretionary
expense centre it is simply the difference between the budgeted input and the actual input. It in no way
measures the value of the output, if actual expense do not exceed the budget amount, the manager has
‘lived within the budget ‘ however ,because by definition the budget does not purport to measure the
optimum amount of spending we cannot say that living within the budget is efficient performance .
Differences in budgeting perspective of engineered and discretionary expense centre

Budget preparation

The decision that management make about a discretionary expense budget are different from the
decisions that it makes about the budget for an engineered expense center. For the latter, management
decides whether the proposed operating budget represent the cost of performing task efficiently for the
coming period. Management is not so much concerned with the magnitude of the task because this is
largely determined by the actions of other responsibility centers, such as the marketing departments’
ability to generate sales. In formulating the budget for a discretionary expense center, however
management principal task is to decide on the magnitude of the job that should be done.

Incremental budgeting

Here the current level expenses in a discretionary expense center is taken as a starting points this
amount is adjusted for inflation for anticipated changes in the workload of continuing tasks for special
tasks and if the data are readily available for the cost of comparable work in similar units.
There are two drawbacks to incremental budgeting. First because managers of these centers typically
want to provide more service they tend to request additional resources in the budgeting process and if
they make a sufficiently strong case these request will be granted. This tendency is expressed in
Parkinson’s second law: overhead costs tend to increase period. There is ample evidence that not all
this upward creep in cost is necessary.

This problem is especially compounded by the fact that the current level of expenditure in the
discretionary expenses center is taken for granted and is not re-examined during the budget preparation
process. Second when a company faces a crises or when a new management takes over overhead costs
are sometimes drastically reduced without any adverse consequences.

Despite this limitation most budgeting in discretionary expense centers is incremental. Time does not
permit the more thorough analysis described in the next section.

Zero based review

An alternative approach is to make a thorough analysis of each discretionary expense center on a


schedule that will cover all of them over a period of five year or so. That analysis provides a new base.
There is a likelihood that expenses will creep up gradually over the next five years and this is tolerated
at the end of five years, another new base is established. Such an analysis is often called a zero base
review.

15
In contrast with incremental budgeting which takes the current level of spending as the starting point
this more intensive review attempts to build up de now the resources that actually are needed by the
activity. Basic question are raised;(1) should use customer?(2) what should the quality level be ?are
we doing too much(3)should the function be performed in this way (4) how much should it cost?

Cost variability

In discretionary expense center costs tend to vary with volume from one year to the next but they tend
not to vary with short run fluctuation in volume within a given year. By contrast costs in engineering
expense center are expected to vary with short run changes in volume. In part this reflect the fact that
volume changes do have an impact throughout the company even though their actual impact cannot be
measures the ; in part this reflect the fact that volume changes do have an impact throughout the
company even though their actual impact cannot be measured in part this result from a management
personnel and personnel related costs are by far the largest expense item in most discretionary expense
center the annual budget for these center tend to be a constant percentage of budgeted sales volume.

Explain some factors which may influence top management style and the implication of the
top management style on management control.

The management control function in an organization is influenced by the style of senior management.
The style of the chief executive officer affects the management control process in the entire
organization. Similarly, the style of the business unit manager affects the unit's management control
process, and the style of functional department managers affects the management control process in
their functional areas.

Differences in Management Styles

Managers manage differently. Some rely heavily on reports and certain formal documents; others
prefer conversations and informal contacts. Some are analytical; others use trial and error. Some are
risk takers; others are risk averse. Some are process oriented; others are results oriented. Some are
long-term oriented; others are short-term oriented. Some emphasize monetary rewards; others
emphasize a broader set of rewards.

Management style is influenced by the manager's background and personality. Background includes
things like age, formal education, and experience in a given function, such as manufacturing,
technology, marketing, or finance. Personality characteristics include such variables as the manager's
willingness to take risks and his or her tolerance for ambiguity.

Implications for Management Control

The various dimensions of management style significantly influence the operation of the control
systems. Even if the same reports with the same set of data go with the same frequency to the CEO,
two CEOs with different styles would use these reports very differently to manage the business units.

Style affects the management control process – how the CEO prefers to use the information, conducts
performance review meetings, and so on – which in turn affects how the control system actually

16
operates, even if the formal structure does not change under a new CEO. In fact, when CEOs change,
subordinates typically infer what the new CEO really wants based on how he or she interacts during
the management control process.

Personal versus Impersonal Controls

Presence of personal versus impersonal controls in organizations is an aspect of managerial style.


Managers differ on how much importance they attach to formal budgets and reports as well as informal
conversations and other personal contacts. Some managers are "numbers oriented"; they want a large
flow of quantitative information, and they spend much time analyzing this information and deriving
tentative conclusions from it. Other managers are "people oriented"; they look at a few numbers, but
they usually arrive at their conclusions by talking with people, judging the relevance and importance
of what they learn partly on their appraisal of the other person.

They visit various locations and spend time talking with both supervisors and staff to get a sense of
how well things are going.

Managers' attitudes toward formal reports affect the amount of detail they want, the frequency of these
reports, and even their preference for graphs rather than tables of numbers, and whether they want
numerical reports supplemented with written comments. Designers of management control systems
need to identify these preferences and accommodate them.

Tight versus Loose Controls

A manager's style affects the degree of tight versus loose control in any situation. The manager of a
routine production responsibility center can be controlled relatively tightly or loosely, and the actual
control reflects the style of the manager's superior. Thus, the degree of tightness or looseness often is
not revealed by the content of the forms or aspects of the formal control documents, rules, or
procedures. It is a factor of how these formal devices are used.

The degree of looseness tends to increase at successively higher levels in the organization hierarchy:
higher-level managers typically tend to pay less attention to details and more to overall results.

The style of the CEO has a profound impact on management control. If a new senior manager with a
different style takes over, the system tends to change correspondingly. It might happen that the
manager's style is not a good fit with the organization's management control requirements. If the
manager recognizes this incongruity and adapts his or her style accordingly, the problem disappears.
If, however, the manager is unwilling or unable to change, the organization will experience
performance problems. The solution in this case might be to change the manager.

Explain advantages and disadvantages of two step transfer pricing and profit sharing methods

Transfer pricing

If two or more profit center is jointly responsible for product development manufacturing and
marketing each should share in the revenue that is generated when the product is finally sold. The
transfer price is not primarily an accounting tool; rather, it is a behavioral tool that motivates manager

17
to make the right decisions. In particular the transfer price should be designed so that it accomplishes
the following objective:
It should provide each segment with the relevant information required to determine the optimum
tradeoff between company cost and revenues

It should induce goal congruent decisions that is the system should be so designed that decision
improve business unit to earn more profit

It should help measure the economic performance of the individual profit center

Two step pricing

First, a charge is made for each unit sold that is equal to the standard variable cost of production.
Second a periodic charge is made for the buying unit. One or both of these components should include
a profit margin.

The two step pricing method correct this problem by transferring variable cost on a per unit basis, and
transferring fixed cost and profit on a lump sum basis under this method the transfer price for product
A would be 5$ for each unit that unit Y purchases plus $20000 per month for fixed cost. Plus $10000
per month for profit: if transfer of product A in a certain month are at the expected amount 5000 units
then under the two step method unit y will pay the variable cost of $25000 plus $30000 for the fixed
cost and profit a total of $55000 .this is the same amount as the amount it would pay unit x if the
transfer price is less than 5000 units say 4000unoits.unit y would pay $50000 under the two step
methods compared with the $44000 it would pay if the transfer price were $11 per unit. The difference
is their transfer prices were for not using a portion of unit X capacity that it has reserved.

Note that under two step method the company variable cost for product A is identifiable to unit Y
variable cost for the product, and unit Y will make the correct short term marketing decisions. Unit Y
also has information on upstream fixed costs and profit related to product A and it can use these data
for long term decision.

The fixed cost calculation in the two step pricing method is based on the capacity that is reserved for
the production of product A that is sold to unit Y the investment represented by this capacity is
allocated to product A. The return on investment that unit X earns on competitive product is calculated
and multiplied by the investment assigned to the product.

In the example we calculated the profit allowance as a fixed monthly amount. It would be appropriate
under some circumstance to divide the investment into variable and fixed component. Then, a profit
allowance based on a return on investment on variable assets would be added to the standard variable
cost for each unit sold.

Profit sharing

If the two step pricing system just described is not feasible, a profit sharing system might be used to
ensure congruence of business unit interest with company interest. This system operates somewhat as
follows.

1. The product is transferred to the marketing unit at standard variable cost.


2. After the product is sold, the business units share the contribution earned which is selling price
minus the variable manufacturing and marketing costs.

18
This method of pricing may be appropriate if the demand for the manufactured product is not steady
enough to warrant the permanent assignment of facilities as in the two step method. In general, this
method accomplished the purpose of making the marketing unit’s interest congruent with the
companies. There are several practical problems in implementing such profit sharing system. First,
there can be arguments over the way contribution is divided between the two profit centers. Which is
costly, time consuming and work against basic reason for decentralization namely autonomy of the
business units mangers. Second, arbitrarily divided up the profit between units does not give valid
information on the profitability of each segment of the organization.

Third since the contribution is not allocated until after the sale has been made the manufacturing units
contribution depends upon the marketing unit’s ability to sell and on the actual selling price.
Manufacturing units may perceive this situation to be unfair

Two set of price

In this method, the manufacturing unit’s revenue is credited at the outside sales price, and the buying
unit is charged the total standard costs. The difference is changed to a headquarter account and
eliminated when the business unit statement are consolidated, this transfer pricing method is
sometimes used when there are frequent conflict between the buying and selling units that cannot be
resolved by one of the other method both the buying and selling

There are several disadvantages to the system of having two set of transaction prices,
however the sum of the business unit profit is greater than overall company profits, senior
management must be aware of this situation in approved budget for the business units and in
subsequent evaluation of performance against these budget. Also, this system create an illusion
feeling that business units are making money while in fact the overall company might be losing
after taking account of the debits to headquarter. Further this system might motivate business unit
to concentrate more on internal transfers at the expense of outside sales

The fact that the conflict between the business units would be lessened under this system
could be viewed as a weakness. Sometime, it is better for the headquarter to be aware of the
conflict arising out of transfer prices because such conflict may signal problem in either the
organizational structure or In other management systems. Under the two sets of prices method
these conflicts are smoothed over thereby not alerting senior management to these problems.

Discuss special challenges faced in controlling R & D activities and possible management
initiatives

Type of financial control:

The financial control exercised in a discretionary expense center is quite different from that in
engineered center the latter attempts to minimize operating cost by setting a standard and reporting
actual costs against this standards. The main purpose of a discretionary expense budget on the other
hand is to allow the manager to control Cost for particular in the planning. Costs are controlled
primarily by deciding what task should be undertaken and what level of effort is appropriate for each.

19
Thus in a discretionary expense center financial control is primary exercised at the planning stage
before the amount are incurred.

Measurement of performance:

The primary job of the manager of a discretionary expense center is to accomplish the desired output
spending an amount that is on budget is satisfactory. This is in contrast with the report in an
engineered expense center which helps higher management to evaluate the manger efficiency. If these
two types of responsibility center are carefully distinguished management may treat the performance
report for the discretionary expense center as if it were an indication of efficiency Control over
spending can be exercised by requiring that the manger approved be obtain before the budget is over
sometimes a certain percentage of overrun is permitted without additional approval if the budget really
set forth the best estimate of actual cost there is 50 percent probability that it will overrun and this is
the reason that some latitude is often permitted.

Control problems:

The control of R & D centers, which are also discretionary expense center is difficult for the following
at least a semi tangible output reasons.

1. Results are difficult to measure quantitatively. As contrasted with administrative activities, R&D
usually has at least a semi tangible output in patent, new products, or new processes. Nevertheless,
the relationship of these outputs to inputs is difficult to measure and appraise. A complete product
of an R&D group may require several year of effort; consequently input as stated in an annual
budget may be unrelated to outputs.

Even if an output can be identified a reliable estimate of its value often cannot be made. Even if
the value of the output can be calculated, it is usually not possible for management to evaluate the
efficiency of the R&D effort because of its technical nature. A brilliant effort may come up against
an insuperable obstacle, whereas a mediocre effort may, by luck result in a bonanza.

2. The goal congruence problem in R&D center is similar to that in administrative centers. The
research managers typically want to build the best research organization that money can buy, even
though this is more expensive than the company can afford. A further problem is that research
people often may not have sufficient knowledge of the business to determine the optimum
direction of the research efforts.

3. Research and development can seldom be controlled effectively on an annual basis. A research
project may take year s to reach fruition, and the organization must be built up slowly over a long
time period. The principal cost is for the work force obtaining highly skilled scientific talented is
often difficult, and short term fluctuation in the work force are in efficient. It is not reasonable,
therefore to reduce R&D costs in years when profits are low and increase them in year when
profits are high. R&D should be looked at as a long term investment not as an activity that varies
with short run corporate profitability.

The R&D continuum:

20
Activities conducted by R&D organization lie along a continuum. At one extreme is basic research; the
other extreme is product testing. Basic research has two characteristics: first, it is unplanned
management at most can specify the general area that is to be explored second there is often a very
long time lag before basic research result in successful new product introductions.

Financial control system has little value in managing basic research activities. In some companies,
basic research in included as a lump sum in the research program and budget. In others, no specific
allowance is made for basic research as such; there is an understanding that scientists and engineers
can devote part of their time to explorations in whatever direction they find most interesting, subject
only to informal agreement with their supervisor.

For product testing projects, on the other hand, the time and financial requirement can be estimated,
not as accurately as production activities.

Explain problems faced in pricing corporate services provided to business units organized as
Profit Centers

Services are intangible in nature. This characteristic of services makes it difficult for pricing. Charging
business units for services furnished by corporate staff units becomes challenging work due to
intangibility of services. While pricing corporate services, we exclude the cost of central service staff
units over which business units have no control (e.g., central accounting, public relations, and
administration). If these costs are charged at all, they are allocated, and the allocations do not include
a profit component. The allocations are not transfer prices.

We need to consider two types of transfers:

• For central services that the receiving unit must accept but can at least partially control the amount
used.
• For central services that the business unit can decide whether or not to use.

Business units may be required to use company staffs for services such as information technology and
research and development. In these situations, the business unit manager cannot control the efficiency
with which these activities are performed but can control the amount of the service received. There are
three schools of thought about such services.

One school holds that a business unit should pay the standard variable cost of the discretionary
services. If it pays less than this, it will be motivated to use more of the service than is economically
justified. On the other hand, if business unit managers are required to pay more than the variable cost,
they might not elect to use certain services that senior management believes worthwhile from the
company's viewpoint. This possibility is most likely when senior management introduces a new
service, such as a new project analysis program. The low price is analogous to the introductory price
that companies sometimes use for new products.

A second school of thought advocates a price equal to the standard variable cost plus a fair share of the
standard fixed costs-that is, the full cost. Proponents argue that if the business units do not believe the
services are worth at least this amount, something is wrong with either the quality or the efficiency of
the service unit. Full cost represents the company's long run costs, and this is the amount that should
be paid.

21
A third school advocates a price that is equivalent to the market price, or to standard full cost plus a
profit margin. The market price would be used if available (e.g., costs charged by a computer service
bureau); if not, the price would be full cost plus a return on investment. The rationale for this position
is that the capital employed by service units should earn a return just as the capital employed by
manufacturing units does. Also, the business units would incur the investment if they provided their
own service.

Optional Use of Services

In some cases, management may decide that business units can choose whether to use central service
units. Business units may procure the service from outside, develop their own capability, or choose not
to use the service at all. This type of arrangement is most often found for such activities as information
technology, internal consulting groups, and maintenance work. These service centers are independent;
they must stand on their own feet. If the internal services are not competitive with outside providers,
the scope of their activity will be contracted or their services may be outsourced completely.

For example, Commodore Business Machines outsourced one of its central service activities-customer
service-to Federal Express. James Reeder, Commodore's vice president of customer satisfaction, said,
"At that time we didn't have the greatest reputation for customer service and satisfaction. But this was
FedEx's specialty, handling more than 300,000 calls for service each day. Commodore arranged for
FedEx to handle the entire telephone customer service operation from FedEx's hub in Memphis.

After losing $29 million online the previous year, Borders Group turned to rival Amazon.com to
manage its online sales. Borders get to maintain an Internet sales channel and gains the operational
effectiveness provided by Amazon.com while being able to focus on the growth of its bricks and
mortar business.

In this situation, business unit managers control both the amount and the efficiency of the central
services. Under these conditions, these central groups are profit centers. Their transfer prices should
be based on the same considerations as those governing other transfer prices.

(Numerical) MCS – 2004

Division B of Shayana company contracted to buy from Div. A, 20,000 units of a components
which goes into the final product made by Div. B. The transfer price for this internal transaction
was set at Rs. 120 per unit by mutual agreement. This comprises of (per unit) Direct and
Variable labour cost of Rs. 20; Material Cost of Rs.60; Fixed overheads of Rs.20 (lumpsum Rs.4
lacs) and Rs.20 lacs that Div. A would require for this additional activity. During the year, actual
off take of Div. B from Div. A was 19,600 units. Div. A was able to reduce material consumption
by 5% but its budgeted investment overshot by 10%.

a) As Financial controller of Div. A, compare Actual Vs Budgetred Performance

22
b) Its implications for Management Control?

Solution:
a)
Particulars For 19,600
20,000Budgeted Actual Actual
Units (Total in Rs.) (Rs. Per Unit) (Total in Rs.)
Budgeted
Direct and (Rs.
20 Per 4,00,000 20 3,92,000
Variable Labour
Cost
Material Cost 60 12,00,000 57 11,17,200
Fixed Overheads 20 4,00,000 4,00,000
Total Cost 100 20,00,000 19,09,200
Transfer Price 120 24,00,000 119.86 23,49,200
Profit 20 4,00,000 4,40,000
Investment 20 20,00,000 22,00,000
ROI = 20% 20%
Profit/Investment

Despite of increase in investment by 10%, there is negligible difference in transfer price. Also the
sales have decreased by 400 units. Therefore we can say that additional investment has not achieved
any positive results.

23
MCS – 2007

Two Divisions A and B of Satyam Enterprises operate as Profit centers. Division A normally
purchases annually 10,000 nos. of required components from Div. B; which has recently informed
Div. A that it will increase selling price per unit to Rs.1,100. Div. A decided to purchase the
components from open market available at Rs. 1000 per unit. Naturally, Div. B is not happy and
justified its decision to increase price due to inflation and added that overall company
profitability will reduce and the decision will lead to excess capacity in Div. B, whose variable
and fixed costs per unit are respectively Rs. 950 and Rs. 1,100.
a) Assuming that no alternate use exists for excess capacity in Div. B, will company as a whole
benefit if div A buys from the market.
b) If the market price reduces by Rs. 80 per unit. What would be the effect on the company
(assuming Div. B still has excess capacity) if A buys from the market
c) If excess capacity of Div. B could be used for alternative sales at yearly cost savings of Rs.
14.5 lacs, should Div. A purchase from outside?

Justify your answers with figures.


Solution

a) Option A ( Div A buys from outside)


Total Purchase Cost = 10,000 Units * Rs. 1000 = Rs. 1,00,00,000
Total outlay if transferred inside = 10,000 Units * Rs. 950 = Rs. 95,00,000
Since total outlay if transferred inside is lesser than total purchase cost if bought from outside,
relevant cost is the lesser one i.e. Rs. 95,00,000 and overall benefit for the company would be Rs.
5,00,000

b) Option B ( if the market price is reduced by Rs. 80 per unit and A buys from the market)
Total Purchase Cost = 10,000 Units * Rs. 920 = Rs. 92,00,000
Total outlay if transferred inside = 10,000 Units * Rs. 950 = Rs. 95,00,000
Since total purchase cost is lesser than the total outlay if transferred inside, relevant cost is the
lesser one i.e. 92,00,000 and overall benefit for the company would be Rs. 3,00,000

c) Option C ( if excess capacity of Div B could be used for alternative sales at yearly cost
savings of Rs 14.5 lacs, should Div A purchase from outside)
Total Purchase Cost = 10,000 Units * Rs. 1,000 = Rs. 1,00,00,000
Total outlay if transferred inside = 10,000 Units * Rs. 950 = Rs. 95,00,000
Total opportunity cost if transferred inside = Rs. 14,50,000
Total relevant cost becomes Rs. 1,00,00,000
If Div A purchase from outside, overall benefit for the company would be Rs. 9,50,000.
Therefore, Div A should purchase from outside.

Particulars Option A Option B Option C


Amount Amount Amount
Total Purchase Cost 1,00,00,000 92,00,000 1,00,00,000
Total outlay if transferred inside 95,00,000 95,00,000 95,00,000
Total opportunity cost if transferred inside - - 14,50,000
Total relevant cost 95,00,000 92,00,000 1,00,00,000
Net advantage/disadvantage to company as a whole if 5,00,000 (3,00,000) (9,50,000)
it buys from inside

24
Explain the concept of ROI. What are its advantages?

Return on investment (ROI) is the ratio of profit before tax to the gross investment.

ROI is calculated with the help of the following formula:

ROI = (Pre-Tax Profit/Sales) X (Sales/Net Assets) or (Pre-Tax Profits/Net Assets)

The numerator is profit before tax as reported in the P&L account. The profit should include only the profits
arising out of the normal activities of the division. Unusual items of receipts and expenses should be excluded
from the profit figure. One should also ignore windfalls and income from investments not related to the
operations of the division. Tax is excluded from the numerator because the marginal of the SBU is not
responsible for or in control of the tax paid.

Capital employed can be ascertained from the balance sheet by including fixed and current assets. Assets not
currently put to divisional use should be excluded from the investment base. One also needs to exclude their
relative earnings if any. The company should also exclude intangible assets like goodwill, deferred revenue
expenses, preliminary expenses, etc.

ROI can be improved by

a) Increasing the profit margin on sales.


b) Increasing the capital turnover
c) Increasing both profit margin and capital turnover.
d) Reducing cost as that adds to the total earnings of the firm.
e) Increasing the profits by expanding present operations or developing new product line, increasing
market share, etc.
f) Diversifying, introducing productivity imporevement measures, expansion, replacement of old
equipments

Advantages of ROI

a) ROI relates return to the level of investment and not sales as the rate of return is more realistic.
b) ROI can be decomposed into other variables as shown. These variables have tremendous analytical
value.
c) ROI is an effective tool for inter-firm comparison.

25
Many experts regard EVA as a concept superior to ROI and yet in certain cases, EVA does not do
justice to the evaluation of investment center. Explain this phenomenon with as illustration.

EVA does not solve all the problems of measuring profitability in an investment center. In particular, it does not
solve the problem of accounting for fixed assets discussed above unless annuity depreciation is also used, and
this is rarely done in practice. If gross book value is used, a business unit can increase its EVA by taking actions
contrary to the interests of the company, as shown in Exhibit 7-3. If net book value is used, EVA will increase
simply due to the passage of time. Furthermore, EVA will be temporarily depressed by new investments because
of the high net book value in the early years. EVA does solve the problem created by differing profit potentials.
All business units, regardless of profitability, will be motivated to increase investments if the rate of return from
a potential investment exceeds the required rate prescribed by the measurement system.

Moreover, some assets may be undervalued when they are capitalized, and others when they are expensed.
Although the purchase cost of fixed assets is ordinarily capitalized, a substantial amount of investment in start-up
costs, new product development, dealer organization, and so forth may be written off as expenses, and, therefore,
not appear in the investment base. This situation applies especially in marketing units. In these units the
investment amount may be limited to inventories, receivables, and office furniture and equipment. When a group
of units with varying degrees of marketing responsibility are ranked, the unit with the relatively larger marketing
operations will tend to have the highest EVA.

In view of all these problems, some companies have decided to exclude fixed assets from the investment base.
These companies make an interest charge for controllable assets only, and they control fixed assets by separate
devices. Controllable assets are, essentially, receivables and inventory. Business unit management can make day-
to-day decisions that affect the level of these assets. If these decisions are wrong, serious consequences can
occur-quickly. For example, if inventories are too high, unnecessary capital is tied up, and the risk of
obsolescence is increased; whereas, if inventories are too low, production interruptions or lost customer business
can result from the stockouts. To focus attention on these important controllable items, some companies, such as
Quaker Oats, 17 include a capital charge for the items as an element of cost in the business unit income
statement. This acts both to motivate business unit management properly and also to measure the real cost of
resources committed to these items.

Investments in fixed assets are controlled by the capital budgeting process before the fact and by post completion
audits to determine whether the anticipated cash flows, in fact, materialized. This is far from being completely
satisfactory because actual savings or revenues from a fixed asset acquisition may not be identifiable. For
example, if a new machine produces a variety of products, the cost accounting system usually will not identify
the savings attributable to each product.

The argument for evaluating profits and capital investments separately is that this often is consistent with what
senior management wants the business unit manager to accomplish; namely, to obtain the maximum long-run
cash flow from the capital investments the business unit manager controls and to add capital investments only
when they will provide a net return in excess of the company's cost of funding that investment. Investment
decisions, then, are controlled at the point where these decisions are made. Consequently, the capital investment
analysis procedure is of primary importance in investment control. Once the investment has been made, it is
largely a sunk cost and should not influence future decisions. Nevertheless, management wants to know when
capital investment decisions have been made incorrectly, not only because some action may be appropriate with
respect to the person responsible for the mistakes but also because safeguards to prevent a recurrence may be
appropriate.

26
Illustration

What are the different methods to evaluate the performance of an investment centre? Discuss the merits

and demerits of each? Which method would you recommend?

The following techniques are useful in evaluating the performance of an investment centre:

Return on investment (ROI)

The rate of return on investment is determined by dividing net profit or income by the capital employed or
investment made to achieve that profit.

ROI = Profit / Invested capital * 100

ROI consists of two components viz.

27
a. Profit margin
b. Investment turnover

ROI = Net profit / Investment

= (Net profit / Sales) * (Sales / Investment in assets)

It will be seen from the above formula that ROI can be improved by increasing one or both of its components
viz. the profit margin and the investment turnover in any of the following ways:

• Increasing the profit margin


• Increasing the investment turnover
• Increasing both profit margin and investment turnover

Capital employed is taken to be the total of shareholders funds, loans etc

The profit figure used is in calculating ROI is usually taken from the profit and loss account, profit arising out of
the normal activities of the company should only be taken.

Capital employed for the company as a whole can be arrived at as follows:

Share capital of the company xxx


Reserves and surplus xxx
Loans (secured/unsecured) xxx
------
xxx

Less: a. Investment outside the business xxx

b. Preliminary expenses xxx

c. Debit balance of P & L A/c xxx xxx

-------

xxxx

Merits:

Return on investment analysis provides a strong incentive for optimum utilization of the assets of the

company. This encourages managers to obtain assets that will provide a satisfactory return on investment and

to dispose off assets that are not providing an acceptable return. In selecting amongst alternative long-term

investment proposals, ROI provides a suitable measure for assessment of profitability of each proposal.

Demerits:

ROI analysis is not very suitable for short-term projects and performances. In the initial stages a new

investment may yield a small ROI which may mislead the management. Most likely the rate would improve in

course of time when the initial difficulties are overcome.

28
The book value of assets decline due to depreciation, the investment base will continuously decrease in

value, causing the rate of return to increase.

Residual income

Residual income can be defined as the operating profit (or income) of the company less the imputed interest on

the assets used by the company. In other words, interest on the capital invested in the company is treated as a

cost and any surplus is the residual income. Residual income is profit minus notional interest charge on capital

employed.

Residual income is affected by the size of the organization and therefore will not provide a basis for evaluation

of organizational performance. This is probably the main reason why the management continues to make use of

ROI which is relative measure.

Not all projects start off with positive or sufficiently large positive profits in the early years of a project to

produce a positive increment to residual income.

It has been argued that a more suitable measure of performance for investment centres, which could encourage

managers to be more willing to undertake marginally profitable projects, is residual income.

We recommend RI as a method of evaluating performance of an investment centre. Because when RI is adopted

for evaluation purposes, emphasis is placed on marginal profit amount above the cost of capital rather than on

the rate itself.

What are the objectives of Transfer Pricing?

Transfer price if designed appropriately has the following objectives:

It should provide each segment with the relevant information required to determine the optimum trade-off

between company costs and revenues.

• It should induce goal congruent decisions-i.e. the system should be so designed that decisions that

improve business unit profits will also improve company profits.

It should help measure the economic performance of the individual profit centers.

29
• The system should be simple to understand and easy to administer.

What is ideal transfer price in the situations of

a. Limited Market

b. Shortage of Capacity in the industry

The ideal transfer price in the situations of :

a. Limited Market

By limited market it means that the markets for buying and selling profit centers may be limited.

Even in case of limited market the transfer price that is ideal or satisfies the requirement of a profit center

system is the competitive price. In case if a company is not buying or selling its product in an outside market

there are some ways to find the competitive price. They are as follows:

If published market prices are available, they can be used to establish transfer prices. However, these
1.
should be prices actually paid in the market-place and the conditions that exist in the outside market

should be consistent with those existing within the company.

For example, market prices that are applicable to relatively small purchases are not valid in this case.

Market prices are set by bids. This generally can be done only if the low bidder has a reasonable chance of
2.
obtaining the business. One company accomplishes this

– by buying about one-half of a particular group of products outside the company

– and one-half inside the company.

The company then puts all of the products out to bid, but selects one-half to stay inside. The company

obtains valid bids, because low bidders can expect to get some of the business. By contrast, if a company

requests bids solely to obtain a competitive price and does not award the contracts to the low bidder, it

will soon find that either no one bids or that the bids are of questionable value.

If the production profit center sells similar products in outside markets, it is often possible to replicate a
1.
competitive price on the basis of the outside price.

2. If the buying profit center purchases similar products from the outside market, it may be possible to

replicate competitive prices for its proprietary products. This can be done by calculating the cost of the

30
difference in design and other conditions of sale between the competitive products and the proprietary

products.

a. Shortage of Capacity in the industry

In this case, the output of the buying profit center is constrained and again company profits may not be

optimum. Some companies allow either buying profit center to appeal a sourcing decision to a central person or

committee. In this scenario a buying profit center could appeal a selling profit center’s decision to sell outside.

The person/group would then make a sourcing decision on the basis of the company’s best interests. In every

case the transfer price would be the competitive price. In other words, the profit center is appealing only the

sourcing decision.

Even if there are constraints on sourcing, the market price is the best transfer price. If the market price can be

approximated, it is ideal transfer price.

When do you use Cost Based Transfer Pricing?

We use cost-based transfer pricing if there is no way of approximating valid competitive price.

Transfer prices may be set up on the basis of cost plus a profit, even though such transfer prices may be complex

to calculate and the results less satisfactory than a market-based price.

Two aspects need to be considered for cost-based transfer pricing:

The cost basis: The usual basis is the standard cost. Actual costs should not be used because production
1.
inefficiencies will then be passed on to the buying profit center. If the standard costs are used, there is a need

to provide an incentive to set tight standards and to improve standards.

The profit markup: In calculating the profit markup, there also are two decisions:
2.
What is the profit markup to be based?

The simplest and most widely used base is percentage of costs. If this base is used, however, no account is

taken of capital required. A conceptually better base is a percentage of investment. But there may be a major

practical problem in calculating the investment applicable to a given product. If the historical cost of the fixed

assets is used, new facilities designed to reduce prices could actually increase costs because old assets are

undervalued.

31
What is the level of profit allowed?

The second problem with the profit allowance is the amount of the profit. The conceptual solution is to base

the profit allowance on the investment required to meet the volume needed by the buying profit centers. The

investment would be calculated at a “standard” level, with fixed assets and inventories at current replacement

costs. This solution is complicated and, therefore, rarely used in practice.

32
“Transfer Pricing is not an accounting tool” comment with an illustration

If a group has subsidiaries that operate in different countries with different tax rates, manipulating the transfer

prices between the subsidiaries can scale down the overall tax bill of the group. For example the tax rate in

Country A is 20% and is 50% in Country B. In the larger interest of the group, it would be advisable to show

lower profits in Country B and higher profits in Country A. For this, the group can adjust the transfer price in

such a way that the profits in Country A increase and that in Country B get reduced. For this the group should fix

a very high transfer price if the Division in Country A provides goods to the Division in Country B. This will

maximize the profits in Country A and minimize the profits in Country B. The reverse will be true if the Division

in Country A acquires goods from the Division in Country B.

There is also a temptation to set up marketing subsidiaries in countries with low tax rates and transfer products to

them at a relatively low transfer price.

Transfer price is viewed as a major international tax issue. While companies indulge in all types of activities to

lower their tax liability, the tax authorities monitor transfer prices closely in an attempt to collect the full amount

of tax due. For this they enter into agreements whereby tax is paid on specific transactions in one country only.

But if companies set unrealistic transfer price to minimize their tax liabilities and the same is spotted by the tax

authority, then the company is forced to pay tax in both countries leading to double taxation.

There have been instances where companies have fixed unrealistic transfer prices. The first case relates to

Hoffman La Roche that imported two drugs Librium and Valium into UK at prices of 437 pounds and 979

pounds per kilo respectively. While the tax authorities in UK accepted the price, the Monopolies Commission

did not accept the company's argument, since the same drugs were available from an Italian firm for 9 pounds

and 28 pounds per kilo.

The company's lawyers argued the case before the Commission on two grounds viz.

1. The price was not set on cost but on what the market would bear and

2. The company had incurred an R&D cost that was included in the price.

33
These arguments did not go well with the Commission and the company was fined 1.85 million pounds

for the manipulative practices adopted while fixing the transfer price.

The second case is of Nissan. The company had falsely inflated freight charges by 40-60% to reduce the profits.

The manipulation helped the company to hide tax to the tune of 237 million dollars. The next year Nissan was

made to pay 106 million dollars in unpaid tax in the USA because the authorities felt that part of their US

marketing profits were being transferred to Japan, as transfer prices on import of cars and trucks were too high.

Interestingly the Japanese tax authorities took a different view and returned the double tax.

With a view to avoid such cases from recurring, Organisation for Economic Cooperation and Development

issued some guidelines in 1995. These guidelines aim at encouraging world trade. They evolved what came to be

known as the arm's length price. The principle states that the transfer price would be arrived at on the basis as if

the two . companies are independent and unrelated. The price is determined through:

• Comparable Price Method where the price is fixed on the basis of prices of similar products or an

approximation to one.

• Gross Margin Method where a gross margin is established and applied to the seller's manufacturing cost.

In spite of all these efforts, it has to be admitted that setting a fair transfer price is not easy. So the onus

of proving the price has been put on the taxpayer who is required to produce supporting documents. If the

taxpayer fails to do this he is required to pay heavy penalty. For example, in USA, failure to provide

documentary evidence results in a 40% penalty on the arm's length price. In UK the penalty is to the tune of

100% of any tax adjustment. Other countries are also in the process of evolving tight norms for the same.

Countries across the globe also allow the taxpayer to enter into an Advance Pricing Agreement whereby dispute

can be avoided and so also the costly penalty of double taxation and penalty.

Market Price is ideal transfer price even in limited markets. Comments

By limited market it means that the markets for buying and selling profit centers may be limited.

Even in case of limited market the transfer price that is ideal or satisfies the requirement of a profit center

system is the competitive price. In case if a company is not buying or selling its product in an outside market

there are some ways to find the competitive price. They are as follows:

34
1. If published market prices are available, they can be used to establish transfer prices. However, these should

be prices actually paid in the market-place and the conditions that exist in the outside market should be

consistent with those existing within the company. For example, market prices that are applicable to

relatively small purchases are not valid in this case.

2. Market prices are set by bids. This generally can be done only if the low bidder has a reasonable chance of

obtaining the business. One company accomplishes this by buying about one-half of a particular group of

products outside the company and one-half inside the company.

The company then puts all of the products out to bid, but selects one-half to stay inside. The company obtains

valid bids, because low bidders can expect to get some of the business. By contrast, if a company requests bids

solely to obtain a competitive price and does not award the contracts to the low bidder, it will soon find that

either no one bids or that the bids are of questionable value.

3. If the production profit center sells similar products in outside markets, it is often possible to replicate a

competitive price on the basis of the outside price.

4.If the buying profit center purchases similar products from the outside market, it may be possible to replicate

competitive prices for its proprietary products. This can be done by calculating the cost of the difference in

design and other conditions of sale between the competitive products and the proprietary products.So we see

from the above arguments that market price is ideal transfer price even in limited markets

Division of Aparna Company manufactures Product A, which is sold to another division as a component of
its product B; which then is sold to third division to be used as part of its Product C (sold to outside market).
Intra company transactions rule: standard cost plus a 10 percent return on fixed assets and inventory, to be
paid by the buying division.
Standard Cost per Unit Product A Product B Product C

*Purchase of outside material (Rs.) 40 60 20


Direct. Labour (Rs.) 20 20 40
Variable overhead (Rs.) 20 20 40
*Fixed overhead per unit. (Rs.) 60 60 20
Average Inventory (Rs.) 14 lacs 3 lacs 6 lacs
Net Fixed Assets (Rs.) 6 lacs 9 lacs 3.2 lacs
Standard Production (Units) 2 lacs 2 lacs 2 lacs

(a) Determine from above data, transfer prices for Products A, B and Standard Cost of Product C.
(b) Product C could become uncompetitive since upstream margins are added. Comment.

Answer
(a): Standard Cost of Product A
35
Outside material (40 * 2 lac units) 80,00,000
Direct Labour (20 * 2 lac units) 40,00,000
Variable O.H. (20 * 2 lac units) 40,00,000

1,60,00,000
+ 10% on (FA + Inventory)
i.e. 10% on 20 lacs 2,00,000

1,62,00,000

Transfer Price for Product A = 1,62,00,000 = 81


2,00,000

Standard Cost of Product B


Outside material (60 * 2 lac units) 1,20,00,000
Direct Labour (20 * 2 lac units) 40,00,000
Variable O.H. (20 * 2 lac units) 40,00,000

2,00,00,000
+ 10% on (FA + Inventory)
i.e. 10% on 12 lacs 1,20,000

2,01,20,000

Transfer Price for Product A = 2,01,20,000 = 100.6


2,00,000

Standard Cost of Product C

Outside material (20 * 2 lac units) 40,00,000


Direct Labour (40 * 2 lac units) 80,00,000
Variable O.H. (40 * 2 lac units) 80,00,000
Fixed O.H. (20 * 2 lac units) 20,00,000
2,20,00,000

(b) While arriving at the cost of Product C, margins of Product A, which become an input to Product B, and
Product B, which in turn become an input to Product C, are added. So when it is sold to outside market, it
suffers a disadvantage from its competitors as far as pricing is concerned, as its price will normally be high
compared to products of similar category. So it might become uncompetitive.

But in the long run, customers will distinguish between a good product and a bad product and the one with
the best quality will survive. So if the quality of product C is better than its competitors than only it can survive
in this competitive market.

Another strategy for the company is to cut the margins added by Products A and B, and then come out with
Product C with a lower price tag on it. This may do well to the product by making higher revenues and
capturing the market share.

Describe and illustrate significance of human behavior patterns in management control system.

Management control systems influence human behavior. Good management control systems influence
behavior in a goal congruent manner; that is, they ensure that individual actions taken to achieve personal goals
also help to achieve the organization's goals. The concept of goal congruence, describing how it is affected both

36
by informal actions and by formal systems.
Senior management wants the organization to attain the organization's goals. But the individual members of the
organization have their own personal goals, and they are not necessarily consistent with those of the
organization. The central purpose of a management control system, then, is to ensure a high level of what is
called "goal congruence." In a goal congruent process, the actions people are led to take in accordance with
their perceived self interest are also in the best interest of the organization.
The significance of human behavior patterns in management control system can be explained with the help of

Informal Factors that influence Goal Congruence. In the informal forces both internal and external factors

play a key role.

External Factors

External factors are norms of desirable behavior that exist in the society of which the organization is a part.
These norms include a set of attitudes, often collectively referred to as the work ethic, which is manifested in
employees' loyalty to the organization, their diligence, their spirit, and their pride in doing a good job (rather
than just putting in time). Some of these attitudes are local that is, specific to the city or region in which the
organization does its work. In encouraging companies to locate in their city or state, chambers of commerce and
other promotional organizations often claim that their locality has a loyal, diligent workforce. Other attitudes and
norms are industry-specific. Still others are national; some countries, such as Japan and Singapore, have a
reputation for excellent work ethics.

Internal Factors

Culture
The most important internal factor is the organization's own culture-the common beliefs, shared values, norms of
behavior and assumptions that are implicitly and explicitly manifested throughout the organization. Cultural
norms are extremely important since they explain why two organizations with identical formal management
control systems, may vary in terms of actual control. A company's culture usually exists unchanged for many
years. Certain practices become rituals, carried on almost automatically because "this is the way things are done
here." Others are taboo ("we just don't do that here"), although no one may remember why. Organizational
culture is also influenced strongly by the personality and policies of the CEO, and by those of lower-level
managers with respect to the areas they control. If the organization is unionized, the rules and norms accepted by
the union also have a major influence on the organization's culture. Attempts to change practices almost always
meet with resistance, and the larger and more mature the organization, the greater the resistance is.

Management Style
The internal factor that probably has the strongest impact on management control is management style. Usually,
subordinates' attitudes reflect what they perceive their superiors' attitudes to be, and their superiors' attitudes
ultimately stem from the CEO.

Managers come in all shapes and sizes. Some are charismatic and outgoing; others are less ebullient. Some spend
much time looking and talking to people (management by walking around); others rely more heavily on written
reports.

The Informal Organization


The lines on an organization chart depict the formal relationships-that is, the official authority and
responsibilities-of each manager. The chart may show, for example, that the production manager of Division A
reports to the general manager of Division A. But in the course of fulfilling his or her responsibilities, the
production manager of Division A actually communicates with many other people in the organization, as well as
with other managers, support units, the headquarters staff, and people who are simply friends and acquaintances.
In extreme situations, the production manager, with all these other communication sources available, may not
pay adequate attention to messages received from the general manager; this is especially likely to occur when the
production manager is evaluated on production efficiency rather than on overall performance. The realities of the
management control process cannot be understood without recognizing the importance of the relationships that
constitute the informal organization.

37
Perception and Communication
In working toward the goals of the organization, operating managers must know what these goals are and what
actions they are supposed to take in order to achieve them. They receive this information through various
channels, both formal (e.g., budgets and other official documents) and informal (e.g., conversations). Despite
this range of channels, it is not always clear what senior management wants done. An organization is a
complicated entity, and the actions that should be taken by anyone part to further the common goals cannot be
stated with absolute clarity even in the best of circumstances.
Moreover, the messages received from different sources may conflict with one another, or be subject to differing
interpretations. For example, the budget mechanism may convey the impression that managers are supposed to
aim for the highest profits possible in a given year, whereas senior management does not actually want them to
skimp on maintenance or employee training since such actions, although increasing current profits, might reduce
future profitability.

The informal factors discussed above have a major influence on the effectiveness of an organization’s
management control. The other major influence is the formal systems. These systems can be classified into two
types: (1) the management control system itself and (2) rules, which are described in this section.

The Formal Control System

Rules

We use the word rules as shorthand for all types of formal instructions and controls, including: standing
instructions, job descriptions, standard operating procedures, manuals, and ethical guidelines. Rules range from
the most trivial (e.g., paper clips will be issued only on the basis of a signed requisition) to the most
important):e.g., capital expenditures of over $5 million must be approved by the board' of directors).
Some rules are guides; that is, organization members are permitted, and indeed expected, to depart from them,
either under specified circumstances or when their own best judgment indicates that a departure would be in the
best interests of the organization.
Some rules are positive requirements that certain actions be taken (e.g., fire drills at prescribed intervals). Others
are prohibitions against unethical, illegal, or other undesirable actions. Finally, there are rules that should never
be broken under any circumstances: a rule prohibiting the payment of bribes, for example, or a rule that airline
pilots must never take off without permission from the air traffic controller.

Some specific types of rules are listed below:

Physical Controls
Security guards, locked storerooms, vaults, computer passwords, television surveillance, and other physical
controls may be part of the control structure.

Manuals
Much judgment is involved in deciding which rules should be written into a manual, which should be considered
to be guidelines rather than fiats, how much discretion should be allowed, and a host of other considerations.
Manuals in bureaucratic organizations are more detailed than are those in other organizations; large
organizations have more manuals and rules than small ones; centralized organizations have more than
decentralized ones; and organizations with geographically dispersed units performing similar functions (such as
fast-food restaurant chains) have more than do single-site organizations

System Safeguards
Various safeguards are built into the information processing system to ensure that the information flowing
through the system is accurate, and to prevent (or at least minimize) fraud of every sort. These include: cross-
checking totals with details, requiring signatures and other evidence that a transaction has been authorized,
separating duties, counting cash and other portable assets frequently, and a number of other procedures described
in texts on auditing.

Task Control Systems


Task control is the process of assuring that specific tasks are carried out efficiently and effectively. Many of
these tasks are controlled by rules. If a task is automated, the automated system itself provides the control.

38
Concept of profit centre in NPO

By law NPO are allowed to make profit but are restrained from distributing it to owners and
management This way they are non profit making organizations (from the owner's point of view). Such
organizations include religious, charitable and educational trusts. Prime goal of management control systems in
such organization is enhancing the service spread first and if possible then cost control rather and than operating
efficiency. On the financial front, they enjoy many concessions from the government such as taxes, subsidies,
grants etc so also they attract special control from these assisting institutes.

Characteristics:
1. Absent of profit performance measure leads to problems in assessing the efficiency of the organization. If the
organization shows large net income it may be because that NPO may not be providing the services to the
extent possible/ expected. If the organization shows net losses it may show the NPO facing risk of bankruptcy.
Hence non availability of clear-cut performance yardstick makes the problem of control worst.
2. NPO's have contributed capital Plant: NPOs do not have shareholder as its stakeholder. The capital
contribution to the business comes by way of contributions to assets such as building and equipments. Second
kind of contribution could be in the form of monetary assistance, which entitles the organization to reap the
interest on it keeping the principal amount intact.
3. Operating Assets represents the resources used for running day to day activities. And the contributed assets
are not allowed to mix up with the operating assets.

4. Fund accounting: NPO need to keep two types of financial statements one set for contributed capital and
another for operating capital. The nature of the contributed capital is beyond control of the management and
therefore management concentrates on controlling the operating assets/investments.
5. Governance: Usually NPO are managed by trusts, who exercise less control on operational matters. Hence
performance control is less demanding from owners' point of view and difficult from the point of view of
management.

These characteristics pose difficulty in pricing of the product/services - what could be appropriate price? Usually
it is set at total/full cost. The more stress expected on allocation of scare resources. Though not stricter control,
but a sense of control can be built among the managers by way of using budgets for various activities and
expenses. Non profit basis makes performance evaluation quite impossible. But one can make the things easier
by concentrating on adherence to costs budgets, and enhancing the service base.

Management control in matrix structures

Matrix organizational structure assigns multiple responsibilities to the functional heads. Evaluation of
performance of such organizational entities is very difficult. Though they offer economies of using scares
functional staff, it poses problems of casting the individual responsibility. This form of organization is very
complex, from the point of view of management control system.
At the end we must not forget that the management control system is for the organization and not the
organization exists for management control system. One has to mold and remold the management control system
to suit the given organization structure

39
A citation by Anthony is worth noting in this regard.

Usually in an advertisement agency, account supervisors are shifted from one


account to another on periodic basis, this practice allows the agency to look at the account from the perspectives
of different executives. However taking in to consideration the time lag of result realization in such services is
quite large. And this may pose problem of performance assessment of a particular executive. This does not mean
a control system designer should insist on abandoning the rotation system of the executives.

Matrix structure offers advantages such as faster decision making process, efficiency and effectiveness but
simultaneously it may pose problems such as added complexity in control function, assignment of responsibility
and authority etc.

Implications of differentiated strategies on controls

Different corporate strategies imply the following differences in the context in which control systems need to be
designed:
As firms become more diversified, corporate-level managers may not have significant knowledge of, or
experience in, the activities of the company's various business units. If so, corporate-level managers for highly
diversified firms cannot expect to control the different businesses on the basis of intimate knowledge of their
activities, and performance evaluation tends to be carried out at arm's length.

Single-industry and related diversified firms possess corporatewide core competencies (on which the strategies
of most of the business units are based. Communication channels and transfer of competencies across business
units, therefore, are critical in such firms. In contrast, there are low levels of interdependence among the business
units of unrelated diversified firms. This implies that as firms become more diversified, it may be desirable to
change the balance in control systems from an emphasis on fostering cooperation to an emphasis on encouraging
entrepreneurial spirit.

Strategic planning:

Given the low level of interdependencies, conglomerates tend to use vertical strategic planning systems-that is,
business units prepare strategic plans & submit to senior management to review & approve. The horizontal
dimension might be incorporated into the strategic planning process in a number of different ways. First, a group
executive might be given the responsibility to develop a strategic plan for the group as a whole that explicitly
identifies synergies across individual business units within the group. Second, strategic plans of individual
business units could have an interdependence section, in which the general manager of the business unit
identifies the focal linkages with other business units and how those linkages will be exploited. Third, the
corporate office could require joint strategic plans for interdependent business units. Finally, strategic plans of
individual business units could be circulated to managers of similar business units to critique and review.

These methods are not mutually exclusive. In fact, several of them could be pursued fruitfully at the same time.

Budgeting

The chief executives of single-industry firms may be able to control the operations of subordinates through
informal and personally oriented mechanisms, such as frequent personal interactions. This lessens the need to
rely as heavily on the budgeting system as the tool of control.
On the other hand, in a conglomerate it is nearly impossible for the chief executive to rely on informal
interpersonal interactions as a control tool; much of the communication and control has to be achieved through
the formal budgeting stem. This implies the following budgeting system characteristics in a conglomerate.
Business unit managers have somewhat greater influence in developing their budgets since they, not the
corporate office, possess most of the information about their respective product/market environments. Greater
emphasis is often placed on meeting the budget since the chief executive has no other informal controls
available.
40
Transfer Pricing

Transfers of goods and services between business units are more frequent in single-industry and related
diversified firms than in conglomerates. The usual transfer pricing policy in a conglomerate is to give sourcing
flexibility to business units and use arm's-length market prices. However, in a single-industry or a related
diversified firm, synergies may be important, and business units may not be given the freedom to make sourcing
decisions.

Incentive Compensation

The incentive compensation policy tends to differ across corporate strategies in the following ways-

Use of formulas:
Conglomerates, in general, are more likely to use formulas to determine business unit managers' bonuses; that
is, they may base a larger portion of the bonus on quantitative, financial measures, such as X percent bonus on
actual economic value added (EVA) in excess of budgeted EVA. These formula-based bonus plans are employed
because senior management typically is not familiar with what goes on in a variety of disparate businesses.
Senior managers of single-industry and related diversified firms tend to base a larger fraction of the business unit
managers’ bonus on subjective factors. In many related diversified firms, greater degrees of interrelationships
imply that one unit's performance can be affected by the decisions and actions of other units. Therefore, for
companies with highly interdependent business units, formula-based plans that are tied strictly to financial
performance criteria could be counterproductive.

Profitability measures:
In the case of unrelated diversified firms, the incentive bonus of the 'business unit managers tend to be
determined primarily by the profitabi1ity of that unit, rather than the profitability of the firm~ Its purpose is to
motivate managers to act as though the business unit were their own company.
In contrast, single-industry and related diversified firms tend to base the incentive bonus of a business unit
manager on both the performance of that unit and the performance of a larger organizational unit (such as the
product group to which the business unit belongs or perhaps even .the overall corporation). When business units
are interdependent, the more the incentive bonus of general managers emphasizes the separate performance of
each unit, the greater the possibility of interunit conflict. On the other hand, basing the bonus of general
managers more on the overall corporate performance is likely to encourage greater interunit cooperation, thereby
increasing managers' motivation to exploit interdependencies rather than their individual results.

Business Unit Strategy:


Diversified corporations segment themselves into business units and typically assign different strategies to the
individual business units. Many chief executive officers of multi business organizations do not adopt a
standardized, uniform approach to controlling their business units; instead, they tailor the approach to each
business unit's strategy.
The strategy of a business unit depends on two interrelated aspects:
(1) Its mission ("What are its overall objectives?") and (2) its competitive advantage.
("How should the business unit compete in its industry to accomplish its mission?"). Typically
business units choose from four missions: build, hold, harvest, and divest. The business unit has two generic
ways to compete and develop a sustainable competitive advantage: low cost and differentiation.

Mission
The mission for existing business units could be either build, hold, or harvest. These missions constitute a
continuum, with "pure build" at one end and "pure harvest" at the other end. To implement the strategy
effectively, there should be congruence between the mission chosen and the types of controls used. The mission
of the business unit influences the uncertainties that general managers face and the short-term versus long-term
trade-offs they make.
Management control systems can be systematically varied to help motivate the manager to cope effectively with
41
uncertainty and make appropriate short-term versus long term trade-offs. Thus, different missions often require
systematically different management control systems.

Mission and Uncertainty

"Build" units tend to face greater environmental uncertainty than "harvest" units for several
reasons: Build strategies typically are undertaken in the growth stage of the product life cycle, whereas harvest
strategies typically are undertaken in the mature decline stage of the product life cycle. Such factors as
manufacturing process; product technology; market demand; relations with suppliers, buyers, and distribution
channels; number of competitors; and competitive structure change more rapidly and are more unpredictable in
the growth stage than in the mature/decline stage.
An objective of a build business unit is to increase market share. Because the total market share of all firms in an
industry is 100 percent, the .battle for market share is a zero-sum game; thus, a build strategy puts a business unit
in greater conflict with its competitors than does a harvest strategy. Competitors' actions are likely to be
unpredictable, and this contributes to the uncertainty that build business units face.

On both the input side and the output side, build managers tend to experience greater dependencies on external
individuals and organizations than do harvest managers. For instance, a build mission signifies additional capital
investment (greater dependence on capital markets), expansion of capacity (greater dependence on the
technological environment), increase in market share (greater dependence on customers and competitors),
increase in production volume (greater dependence on raw material suppliers and labor markets), and so on. The
greater the external dependencies a business unit faces, the greater the uncertainty it confronts.
.
Build business units are often in new and evolving industries; thus, build managers are likely to have less
experience in their industries. This also contributes to the greater uncertainty that managers of build units face in
dealing with external constituencies.

Mission and Time Span

The choice of build versus harvest strategies has implications for short-term versus long-term profit trade-offs.
The share-building strategy includes (a) price 'cutting, (b) major R&D expenditures (to introduce new products),
and (c) major market development expenditures. These actions are aimed at establishing market leadership, but
they depress short-term profits. Thus, many decisions that a build unit manager makes, today may not result in
profits until some future period. A harvest strategy, on the other hand, concentrates on maximizing short-term
profits.

Strategic Planning

When the environment is uncertain, the strategic planning process is especially important management needs to
think about how to cope with the uncertainties, and this usually requll1 longer-range view of planning than is
possible in the annual budget. If the environment is stable, there may be no strategic planning process at all or
only a broad-brush strategic plan. Thus, the strategic planning process is more critical and more important for
build, as compared with harvest, business units. Nevertheless, some strategic planning of the harvest business
units may be necessary because the company's overall strategic plan must encompass all of its businesses to
effectively balance cash flows.
In screening capital investments and allocating resources, the system may be more quantitative and financial for
harvest units. A harvest business unit operates in a mature industry and does not offer tremendous new
investment possibilities. Hence, the required earnings rate for such a business unit may be relatively high to
motivate the manager to search for project with truly exceptional returns. Because harvest units tend to
experience stable environments with predictable products, technologies, competitors, and customers), discounted
cash flow PCF) analysis often can be used more confidently. The required information used to evaluate
investments from harvest units is primarily financial.

A build unit, however, is positioned on the growth stage of the product life cycle. Since the corporate office
wants to take advantage of the opportunities in a growing market, senior management may set a relatively low
discount rate, thereby motivating build managers to forward more investment ideas to corporate office. Given the
product/market uncertainties, financial analysis of some projects from build units may be unreliable. For such
projects, nonfinancial data are more important.

42
Budgeting

The calculational aspects of variance analysis comparing actual results with the budget identify variances as
either favorable or unfavorable. However, a favorable variance does not necessarily imply favorable
performance, nor does an unfavorable variance imply unfavorable performance. The link between a favorable or
unfavorable variance, on the one hand, and favorable or unfavorable performance, on the other hand, depends on
the strategic context of the business unit under evaluation.

Incentive Compensation System

In designing an incentive compensation package for business unit managers, the following questions need to be
resolved:
1. What should the size of incentive bonus payments be relative to the general manager's base salary?
Should the incentive bonus payments have upper limits?
2. What measures of performance (e.g., profit, EVA, sales volume, market share, product
development) should be used when deciding the general manager's incentive bonus awards? If multiple
performance measures are employed, how should they be weighted?
3. How much reliance should be placed on subjective judgments in deciding on the bonus amount?
4. How frequently (semiannual, annual, biennial, etc.) should incentive awards be made?

With respect to the first question, many firms use the principle that the riskier the strategy, the greater the
proportion of the general manager's compensation in bonus compared to salary (the "risk/return" principle). They
maintain that because managers in charge of more uncertain task situations should be willing.to take greater
risks, they should have a higher percentage of their remuneration in the form of an incentive bonus. Thus,
"build" managers are more likely than "harvest" managers to rely on bonuses.
As to the second question, when rewards are tied to certain performance criteria, behaviour ls influenced by the
desire to optimize performance with respect to those criteria. Some performance criteria (cost control, operating
profits, and cash flow from operations) focus more on short-term results, whereas other performance criteria
(market share, new product development, market development, and people development) focus on long-term
profitability. Thus,
linking incentive bonus to short-term criteria tends to promote a short-term focus on the part of the general
manager and, similarly, linking incentive bonus to long-term criteria is likely to promote long-term focus.
Considering the relative differences in time horizons of build and harvest managers, it may not be appropriate to
use a single, uniform financial criterion, such as operating profits, to evaluate the performance of every business
unit. A better idea would be louse multiple performance criteria, with differential weights for each criterion
depending on the business unit's mission.
The third question asks how much subjective judgment should affect bonus amounts. At one extreme, a
manager's bonus might be a strict formula-based plan, with the bonus tied to performance on quantifiable criteria
(e.g., X percent bonus on actual profits in excess of budgeted profits). At the other extreme, a manager's
incentive bonus amounts might be based solely on the superior's subjective judgment or discretion. Alternatively,
incentive bonus amounts might also be based on a combination of formula-based and subjective approaches.
Performance on most long-term criteria (market development, new-product development, and people
development) is harder to measure objectively than is performance along most short-run criteria (operating
profits, cash flow from operations, and return on investment).As already noted, build managers- in contrast with
harvest managers, should concentrate more on the long run, so they typically are evaluated more subjectively
than are harvest managers.

As to the fourth question, the frequency of bonus awards does influence the time horizon of managers. More
frequent bonus awards encourage managers to concentrate on short-term performance since they have the effect
of motivating managers to focus on those facets of the business they can affect in the short run.

Competitive Advantage
A business unit can choose to compete. Either as a differentiated player or as a low-cost player, Choosing a
differentiation 'approach, rather than a low-cost approach, increases uncertainty of a business unit's task
environment for three reasons.
43
1. Product innovation is more critical for differentiation business units than for low cost business units.
This is partly because a low-cost business unit, with primary emphasis on cost reduction, typically
prefers to keep its product offerings stable over time; a differentiation business unit, with its primary
focus on uniqueness & exclusivity, is likely to engage in greater product innovation.

2. A low cost business unit typically tend to have narrow product lines to minimize the inventory carry
costs as well as to benefit from scale economies. Differentiation business units on the other hand tend to
have a broader set of products to create uniqueness.

3. Low cost business units typically produce no-frill commodity products& these products succeed
primarily because they have lower prices than competing products. However product differentiation
business units succeed if customers perceive that the products have advantages over competing
products. Since the customer perception is difficult to learn about, & since customer loyalty is subject to
change resulting from actions of competitors or other reasons, the demand for differentiated products is
typically more difficult to predict than the demand for commodities.

3. Which management control practices, if followed, in performance measurement of investment centres

are likely to induce goal congruence, in respect of following assets:

(i) Cash (ii) Receivables (iii) Inventories (iv) Idle (v) Intangible (vi) Leased

In some business units, the focus is on profit as measured by the difference between revenues and expenses. In
other business units, profit is compared with the assets employed in earning it. We refer to the latter group of
responsibility centers as investment centers.

Measuring Assets Employed

In deciding what investment base to use to evaluate investment center managers, headquarters asks two
questions: First, what practices will induce business unit managers to use their assets most efficiently and to
acquire the proper amount and kind of new assets? Presumably, when their profits are related to assets employed,
business unit managers will try to improve their performance as measured in this way. \Senior management
wants the actions that they take toward this end to be in the best interest of the whole corporation. Second, what
practices best measure the performance of the unit as an economic entity?

Cash
Most companies control cash centrally because central control permits use of a smaller cash balance than
would be the case if each business unit held the cash balances it needed to weather the unevenness of its cash
inflows and outflows. Business unit cash balances may well be only the "float" between daily receipts and daily
disbursements. Consequently, the actual cash balances at the business unit level tend to be much smaller than
would be required if the business unit were an independent company. Many companies therefore use a formula
to calculate the cash to be included in the investment base. For example, General Motors was reported to use 4.5
percent of annual sales; Du Pont was reported to use two months' costs of sales minus depreciation.

One reason to include cash at a higher amount than the balance carried by a business unit is that the higher
amount is necessary to allow comparisons to outside companies. If only the actual cash were shown: by internal
units would appear abnormally high and might mislead senior management.
Some companies omit cash from the investment base. These companies reason that the amount of cash
approximates the current liabilities; if this is so, the sum of accounts receivable and inventories will approximate
the working capital.

Receivables

Business unit managers can influence the level of receivables, not only indirectly by their ability to
generate sales, and directly, by establishing credit terms by approving individual credit accounts and credit
limits, and by the collecting overdue amount. In the interest of simplicity, receivable included at the actual
44
end-.of-period balances, although the average of intraperiod balances is conceptually a better measure of the
am should be related to profits.

Whether to include accounts receivable at selling prices or at cost of goods sold is debatable. One could
argue that the business unit's real investment in accounts receivable is only the cost of goods sold and that a
satisfactory return on this investment is probably enough. On the other hand, it is possible to argue that the
business unit could reinvest the money collected from accounts receivable, and, therefore, accounts
receivable should be included at selling prices. The usual practice is to take the simpler alternative-that is,
receivables at the book amount, which is the selling price less an allowance for bad debts.

If the business unit does not control credits and collections, receivables may be calculated on a formula
basis. This formula should be consistent with the normal payment period-for example, 30 days' sales where
payment is made 30 days after the shipment of goods.

Inventories

Inventories ordinarily are treated in a manner similar to receivables –that is they are often recorded at end-
of-period amounts even though intraperiod averages would be preferable conceptually. If the company uses
LIFO (last in first out) for financial accounting purposes, a different valuation method usually is used for
business unit profit reporting because LIFO inventory balances tend to be unrealistically low in periods of
inflation. In these circumstances, inventories should be valued at standard or average costs, and these same
costs should be used to measure cost of sales on the business unit income statement

If work-in-process inventory is financed by advance payments or by progress payments from the customer,
as is typically the case with goods that require a long manufacturing period, these payments either are subtracted
from the gross inventory amounts or reported as liabilities.

For e.g. with manufacturing periods a year or greater, Boeing received progress payments for its
airplanes and recorded them as liabilities.

Some companies subtract accounts payable from inventory on the grounds that accounts payable represent
financing of part of the inventory by vendors, at zero cost to the business unit. The corporate capital required for
inventories is only the difference between the gross inventory amount and accounts payable. If the business unit
can influence the payment period allowed by vendors, then including accounts payable in the calculation
encourages the manager to seek the most favorable terms. In times of high interest rates or credit stringency,
managers might be encouraged to consider forgoing the cash discount to have, in effect, additional financing
provided by vendors. On the other hand, delaying payments unduly to reduce net current assets may not be in the
company's best interest since this may hurt its credit rating.

Leased Assets

Suppose the business unit whose financial statements are shown in Exhibit 1 (see page 21) sold its fixed
assets for their book value of $300,000, returned the proceeds of the sale to corporate headquarters, and then
leased back the assets at a rental rate of $60,000 per year. As Exhibit 2 (see page 21) shows, the business unit's
income before taxes would decrease because the new rental expense would be higher than the depreciation
charge that was eliminated.

Nevertheless, economic valued added would increase because the higher cost would be more than offset
by the decrease in the capital charge. Because of this, business unit managers are induced to lease, rather than
own, assets whenever the interest charge that is built into the rental cost is less than the capital charge that is
applied to the business unit's investment base. (Here, as elsewhere, this generalization oversimplifies because, in
the real world, the impact of income taxes must also be taken into account.)

Many leases are financing arrangements-that is, they provide an alternative way of getting to use assets that
otherwise would be acquired by funds obtained from debt and equity financing. Financial leases (i.e., long-term
leases equivalent to the present value of the stream of lease charges) are similar to debt and are so reported on
the balance sheet. Financing decisions usually are made by corporate headquarters. For these reasons, restrictions
usually are placed on the business unit manager's freedom to lease assets.
45
Idle Assets

If a business unit has idle asset that can be used by other units, the business unit may be permitted to exclude
them from the investment base if it classifies them as available. The purpose of this permission is to encourage
business unit managers to release underutilized assets to units that may have better use for them. However, if the
fixed assets cannot be used by other units, permitting the business unit manager to remove them from the
investment base could result in dysfunctional actions For example; it could encourage the business unit manager
to idle partially utilized assets that are not earning a return equal to the business unit's profit objective. If there is
no alternative use for the equipment, any contribution from this equipment will improve company profits.

Intangible Assets

Some companies tend to be R&D intensive (e.g., pharmaceutical firms such as Novartis spend huge amounts
on developing new products); others tend to be marketing intensive (e.g., consumer products firms such as
Unilever spend huge amounts on advertising).

There are advantages to capitalizing intangible assets such as R&D and marketing and then amortizing them
over a selected life. This method should change how the business unit manager views these expenditures. By
accounting for these assets as long-term investments, the business unit manager will gain less short-term benefit
from reducing out lays on such item. For instance, if R&D expenditures are expensed immediately, each dollar
of R&D cut would be a dollar more in pretax profits.

On the other hand, if R&D costs are capitalized, each dollar cut will reduce the assets employed by a dollar;
the capital charge is thus reduced only by one dollar times the cost of capital, which has a much smaller positive
impact on economic valued added.

NUMERICAL – ANANYA Ltd. (2004)

Exhibit 2

46
We know that formula for Return on Investment is:

ROI = NET PROFIT

INVESTMENT

Now, Investment = Fixed assets + Net working Capital

(We assume Current Assets as the Net Working Capital as there are no Current Liabilities given in the question)

Therefore, Investment for:

M = 0 + 200 = 200

P = 200 + 1000 = 1200

C = 200 + 500 = 700

Now, Net Profit for M, P and C:

Particulars M P C

Profit before Depreciation & Operating 400 400 400


Expenses

Less- Depreciation (NIL) (100) (50)

Less- Operating Expenses (200) (100) (150)

TOTAL 200 200 200

Therefore,

ROI for:

M = 200 = 100 %

200

P = 200 = 16.67 %

1200

47
C = 200 = 28.57 %

700

Since there are no fixed assets in marketing division, the ROI is higher, but the operating expenses are much
higher for these division.

Hence, any further increase in op exp is likely to drag the ROI down

Since the asset is depreciated for10 years as per SLM method, the depreciation rate is 10 %.

So going ahead if the operating expenses for div P & C remains at the same level, reduction in the value of an
asset due to depreciation is likely to have a positive impact on ROI.

Even the rate of increase in ROI for Div P would be higher since the asset of a higher

value is depreciated than the Div C.

What do you understand by Goal Congruence? What are the informal factors that influence goal
congruence?

This term is used when the same goals are shared by top managers and their subordinates. This is one of
the many criteria used to judge the performance of an accounting system. The system can achieve its goal more
effectively and perform better when organizational goals can be well aligned with the personal and group goals
of subordinates and superiors. The goals of the company should be the same as the goals of the individual
business segments. Corporate goals can be communicated by budgets, organization charts, and job descriptions.

Goal Congruence- Meaning


Individuals work in different hierarchies and handle different responsibilities & may have different goals. But
they must come together as far as Company’s Goal is concerned (there action must speak Co’s language.)

Goal Congruence
Example 1– The HR manager has devised a HR training program to enhance the skills of its sales personnel,
with an objective to enhance their productivity But if company is in strategic need of attaining a certain sales
volume in a given quarter, it can not do so on account of non availability of personnel.

Example 2– The marketing department has planned an impressive advertising campaign, which promises good
returns, But say due to cash crunch Company’s current financial position may not let to lose the strings

Example 3 – Production Manager may get a good applause for reducing cycle time; But at what cost? Building
up the high inventory i.e. higher investment in current assets. While doing so he just overlooked the financial
interest of the company. • After completing the given activity in more efficient manner the concerned manager
scores the point/s on his score card. • Whether his actions are leading to scoring of points on the organization’s
score card too? if it is so then only one can say the organization is marching towards a common goal.

Every individual working in an organization has got his own motive to do the work. Individuals act in their own
interest, based on their own motivations. And it is always not necessarily consistent with the Co’s goal. In a goal
congruence process, the actions the people are led to take in accordance with their perceived self interest are also
in the best interest of the organization i.e. Goal congruence ensures that the action of manager taken in their best
interest is also in the best interest of the organization.

Informal factors that influence goal congruence:

External factors – set of attitudes of the society, work ethics of the society

Internal factors – (Factors within the organization)


• Culture-Common beliefs, shared values, norms of behavior & assumptions
• Implicitly accepted and explicitly built into.
48
• Mgt. Style – Informal/Formal
• The Communication Channels
• Perception and Communication – e.g. Budget (meaning) strict profit.

Organizations with Business Divisions (Profit Centre) format have observed that Divisional Controllers
experience divided loyalty in carrying out their functions, causing a possible dysfunction. How could
such a situation be resolved? Define role of controller which suits your suggestion.

To the extent the decision are decentralized top management may lose some control. Relying on control
reports is not as effective as personal knowledge of an operation. With profit center, top management must
change its approach to control. Instead of personal direction senior management must rely to a considerable
extent on management control reports.

Competent units that were once cooperating as functional units may now compete with one another dis
advantageously. An increase in one manager’s profit may decrease those of another. This decrease in
cooperation may manifest itself in a manager unwillingness to refer sales lead to another business unit, even
though that unit is better qualified to follow up on the lead in production decision that have undesirable cost
consequence on other units or in the hoarding of personnel or equipment that from the overall company
standpoint would be better off used in another units.

There may be too much emphasis on short run profitability at the expense of long run profitability. In the desire
to report high current profits, the profit center manager may skip on R&D, training, maintenance. This tendency
is especially prevalent when the turnover of profit center managers is relatively high. In these circumstances,
manager may have good reason to believe that their action may not affect profitability until after they have
moved to other job.

There is no complete satisfactory system for ensuring that each profit center by optimizing its own profit , will
optimize company profits.

If headquarter management is more capable or has better information then the average profit center manager the
quality of some of the decision may be reduced.

Divisionalization may cause additional cost because it may require additional management staff personnel and
recordkeeping and may lead to redundant at each profit center.
Business units as profit centers:

Business units are usually set up at profit centers. Business unit managers tend to control product development,
manufacturing, and marketing resources. They are in a position to influence revenue and cost and as such can be
held accountable for the bottom line. However as pointed out in the next section a business unit manager
authority may be constrained such constrained should be incorporated in designing and operating profit center.

Constraint on business unit authority

To realize fully the advantage of the profit center concept the business unit manger would have to be as
autonomous as the president of the independent company. As a practical matter however such autonomy is not
feasible. If a company were divided into completely independent units the organization would be giving up the
advantage of size and synergism. Also senior management authority that a board of director gives to the chief
executive. Consequently business unit structure represents trade off between business unit autonomy and
corporate constraint. The effectiveness of a business units organization is largely dependent on how well these
trade off are made.

The performance of a profit center is appraised by comparing actual results for one or more orf these measures
with budgeting amounts. In addition, data on competitors and the industry provide a good cross check on the
appropriate of the budget. Data for individual companies are available from the securities and exchange
commission for about key business ratios; standard & poor computer services, Inc; Robert Morris associates
annual statement studies; and annual survey published in fortune, business week, and Forbes. Trade associations
publish data for the companies in their industries.

Revenues

49
Choosing the appropriate revenue recognition method is important. Should revenue be recognized at the time as
order is received, at the time an order is shipped, or at the time cash is received?

In addition to that decision, issues related to common revenues may need to be considered. There are some
situations in which two or more profit centers participate in the sales effort that results in a sale; ideally, each
should be given appropriate credit for its part in this transaction. Many companies have not given much attention
to the solution of these common revenue problems. They take the position that the identification of price
responsibility for revenue generation is too complicated to be practical and that sale personnel must recognize
they are working not only for their own profit center but also for the overall good of the company. They for
example, may credit the business unit that takes an order for a product handled by the another unit with the
equivalent of a brokerage commission or a finder fee. In the case of a bank the branch performing a service may
be given explicit credit for that service even though the customer account is maintained in another branch.

• Role of controller
• It should publish procedure and forms for the preparation of the budget.
• It should provide assistance to budgetees in the preparation of their budget.
• It should administer the process of making budget revision during the year.
• It should coordinate the work of budget departments in lower echelons
• It should analyze reported performance against budget, interprets the result, and prepares summary report
for senior management.

Part of a multinational group, Sundaram Shoe Company(SSC), established its


own facilities in India over 75 years ago and enjoyed an excellent record-high
market share for its diverse range of shoes, growth and profits. SC markets
its products through company owned shops and its own personnel.
Organization structure is functional. Since 2001, profitability, market share
are slipping. Pressure from cheap Chinese shoes and also premium shoes like
Nike has made the company think< of organizational restructuring and
introducing Comensurate Control System to regain its position. Although SSC
outsources, 30% of products, it is seen as a production oriented company.
SSC wants to adopt measures to reduce costs, strengthen marketing and be
in a position to produce and meet unexpected and unusual customer
demands. How should the company reorganize to achieve Goal Congruence.
Define Performance Metric?

In a goal congruent process, the actions people are led to take in accordance with
their perceived self-interest are also in the best interest of the organization. A firm’s
strategy has a major influence on its structure. The type of structure in turn influences
the design of the organization’s management control systems. Sundaram Shoe
Company’s (SSC) organization structure is functional which involves the notion of a
manager who brings specialized knowledge to bear on decisions related to a specific
function, vis-à-vis a general purpose manager who lacks the specialized knowledge. A
skilled marketing and production manager would be able to make better decisions in their
respective fields. He would also be able to supervise workers in the same function better
than the generalist would. Thus an important advantage of the functional structure is
efficiency. A major disadvantage of this structure is that there is no unambiguous way of
determining the effectiveness of the separate functional managers because each function
contributes jointly to the organization’s final output. Therefore, there is no way of
determining how much of the profit was earned respectively by the several production
departments.

Sundaram Shoe Company which was a market leader for a period of over 75 years
has been losing market share, which has impacted its profitability. Also it needs to be
seen that the company outsources about 30% of its products. The company aims to
strengthen marketing, reduce costs and wants to be in a position to customize products
as per the demands of the customer. Thus, Sundaram needs to re-organize its
50
organization structure which is functional to a Business Unit form of organization. The
benefits of the re-organization would be that the business unit or the division would be
responsible for all the functions involved in producing and marketing a specified product
line. The business managers act almost as if their units are separate companies. They are
responsible for planning and co-coordinating the work of the separate functions. Their
performance is measured by the profitability of the business unit. This is a valid criterion
because profit reflects the activities of both marketing and production.

Though business unit managers exercise broad authority over their units,
headquarters reserves certain key prerogatives. Headquarters are responsible for
obtaining funds for the company as a whole and allocating it to the business unit, as well
as approving budgets and judging the performance of business unit managers, setting
their compensation.

A major advantage of the Business unit structure of organization is that because it


is close to the market for its products than the headquarters, its manager may make
sounder production and marketing decisions than headquarters might and the unit as a
whole reacts to new threats or opportunities quickly. This re-organization would help in
achieving goal congruence in the organization.
Performance Metrics are high-level measures what you are doing; that is, they
assess your overall performance in the areas you are measuring. They are
external in nature and are most closely tied to outputs, customer requirements, and
business needs for the process.
The performance measurement system should cover the following areas at a minimum:

• CUSTOMERS
1. Performance against customer requirements
2. Customer Satisfaction

• PERFORMANCE OF INTERNAL WORK PROCESSES


1. Cycle times
2. Product and service quality
3. Cost performance (could be productivity measures, inventory, etc.) \
• SUPPLIERS
1. Performance of suppliers against your requirements

• FINANCIAL
1. Profitability (could be at the company, product line, or individual level)
2. Market share growth and other standard financial measures

• EMPLOYEE
1. Associate satisfaction

Ananaya & Company comprises of five divisions A, B, C, D and E and the


present performance. metric is return on assets. However, the controller has

51
suggested management to switch over to economic value added (EVA) as the
criterion rather than return on assets. Compute and tabulate both return on
assets and EVA on the basis of following information (Rs. lakhs) and comment
on divisional performance.

Division Profit Fixed Assets Current Assets

--

A 300 800 160

- - ----

B 220 400 1600

C 100 600 1000

________

D 110 400 800

E 180 200 800

Controller feels corporate finance rates on current assets and .fixed assets should be 5%
and 10% respectively.
Solution: Working Note:
Return on Assets = Profit * 100

Total Assets

A = 300/960*100 = 31.25%
B = 220/2000*100 = 11%
C = 100/1600*100 = 6.25%
D = 110/1200*100 = 9.17%
E = 180/1000*100 = 18%
Economic Value Added (EVA) = Profit – (W.A.C.C.* Capital Employed)

In this case,
EVA = Profit – (W.A.C.C. on Fixed Assets * Total Fixed Assets) + (W.A.C.C. on Current
Assets * Total Current Assets)
A = 300 – (0.10*800) + (0.05*160) = 212 lakhs
B = 220 – (0.10*400) + (0.05*1600) = 100 lakhs
C = 100 – (0.10*600) + (0.05*1000) = -10 lakhs
D = 110 – (0.10*400) + (0.05*800) = 30 lakhs
E = 180 – (0.10*200) + (0.05*800) = 120 lakhs
Summary
52
Division Return on Assets (R.O.A.) Economic Value Added (E.V.A.)
(Rs. lakhs)

A 31.25% 212

B 11.00% 100

C 6.25% -10

D 9.17% 30

E 18.00% 120

Comments:
1. It appears from the above analysis that division A has performed the best among the
five divisions.
2. Also, it can be clearly noticed that divisions C and D seem to be in trouble.
3. Division A has performed the best when seen in terms of return on assets and economic
value added.
4. The reason why division A has performed the best is that it has the best working capital
management that can be reflected in the total amount invested in current assets and
which is the least among the five divisions.
5. The above reason holds true for the poor performance of divisions C and D as can be
seen that they have a huge amount invested in current assets which does not indicate
good signs about their operational efficiency.
6. A company which is into an expansion and overall growth mode primarily invests into
fixed assets and this is also one of the major reasons why the performance of division A
is the best amongst all.
7. Though division C has also invested a huge amount in fixed assets the advantage is
offset due to the fact that it perhaps has a larger investment in current assets.
8. Division E is the second best both in terms of R.O.A. as well as E.V.A.
9. Though division E has the same amount invested in current assets as that of division D
and perhaps a lesser amount invested in fixed assets its profitability is much better and
hence it has delivered a better performance.
10. Division B is a better performer than divisions C and D in terms of R.O.A. as well as
E.V.A. but the major problem with this division is that it has a terrible working capital
management. Its current assets are the highest and this reflects that it has huge sums
of money held up either in debtors or inventory or rather it is holding a large amount of
cash which is not a good sign.

Factors which impact service organization

Absence of Inventory Buffer:


Goods can be held as inventory, which is a buffer that dampens the impact on production activity of
fluctuations in sales volume. Services cannot be stored. The airplane seat, hotel room, hospital operating room,
or the hours of lawyers, physicians, scientists, and other professionals that are not used today are gone forever.
Thus, although a manufacturing company can earn revenue in the future from products that are on hand today, a
service company cannot do so. It must try to minimize its unused capacity.

53
Moreover, the costs of many service organizations are essentially fixed in the short run. In the short run, a hotel
cannot reduce its costs substantially by closing off some of its rooms. Accounting firms, law firms, and other
professional organizations are reluctant to layoff professional personnel in times of low sales volume because of
the effect on morale and the costs of rehiring and training.

Difficulty in Controlling Quality:


A manufacturing company can inspect its products before they are shipped to the consumer, and their
quality can be measured visually or with instruments (tolerances, purity, weight, color, and so on). A service
company cannot judge product quality until the moment the service is rendered, and then the judgments are often
subjective. Restaurant management can examine the food in the kitchen, but customer satisfaction depends to a
considerable extent on the way it is served. The quality of education is so difficult to measure that few
educational organizations have a formal quality control system.

Labour Intensive:
Manufacturing companies add equipment and automate production lines, thereby replacing labor and
reducing costs. Most service companies are labor intensive and cannot do this. Hospitals do add expensive
equipment, but mostly to provide better treatment, and this increases costs. A law firm expands by adding
partners and new support personnel.

Multi-Unit Organizations:
Some service organizations operate many units in various locations; each unit relatively small. These
organizations are fast-food restaurant chains, auto rental companies, gasoline service stations, and many others.
Some of the units are owned; others operate under a franchise. The similarity of the separate units provides a
common basis for analyzing budgets and evaluating performance not available to the manufacturing company.
The information for each unit can be compared with system wide or regional averages, and high performers and
low performers can be identified. However because units differ in the mix of services they provide, in the
resources that they use, and in other ways, care must be taken in making such comparisons.

Explain special characteristics of professional organization which would have a bearing on their control
system.

Goals:
A dominant goal of a manufacturing company is to earn a satisfactory profit, specifically a satisfactory
return on assets employed. A professional organization has relatively few tangible assets; its principal asset is the
skill of its professional staff, which doesn't appear on its balance sheet. Return on assets employed, therefore, is
essentially meaningless in such organizations. Their financial goal is to provide adequate compensation to the
professionals.

In many organizations, a related goal is to increase their size. In part, this reflects the natural tendency to
associate success with large size. In part, it reflects economies of scale in using the efforts of a central personnel
staff and units responsible for keeping the organization up to- date. Large public accounting firms need to have
enough local offices to enable them to audit clients who have facilities located throughout the world.

Professionals:
Professional organizations are labor intensive, and the labor is of a special type. Many professionals prefer
to work independently, rather than as part of a team. Professionals who are also managers tend to work only part
time on management activities; senior partners in an accounting firm participate actively in audit engagements;
senior partners in law firms have clients. Education for most professions does not include education in
management, but quite naturally stresses the skills of the profession, rather than management; for this and other
reasons, professionals tend to look down on managers. Professionals tend to give inadequate weight to the
financial implications of their decisions; they want to do the best job they can, re- I regardless of its cost. This
attitude affects the attitude of support staffs and nonprofessionals in the organization; it leads to inadequate cost
control.

Output and Input Measurement:


The output of a professional organization cannot be measured in physical terms, such as units, tons, or
gallons. We can measure the number of hours a lawyer spends on a case, but this is a measure of input, not
output. Output is the effectiveness of the lawyer's work, and this is not measured by the number of pages in a
54
brief or the number of hours in the courtroom. We can measure the number of patients a physician treats in a
day, and even classify these visits by type of complaint; but this is by no means equivalent to measuring the
amount or quality of service the physician has provided. At most, what is measured is the physician's efficiency
in treating patients, which is of some use in identifying slackers and hard workers. Revenues earned is one
measure of output in some professional organizations, but these monetary amounts, at most, relate to the quantity
of services rendered, not to their quality (although poor quality is reflected in reduced revenues in the long run).

Furthermore, the work done by many professionals is non repetitive. No two consulting jobs or research and
development projects are quite the same. This makes it difficult to plan the time required for a task, to set
reasonable standards for task performance, and to judge how satisfactory the performance was. Some tasks are
essentially repetitive: the drafting of simple wills, deeds, sales contracts, and similar documents; the taking of a
physical inventory by an auditor; and certain medical and surgical procedures. The development of standards for
such tasks may be worthwhile, although in using these standards, unusual circumstances that affect a specific job
must be taken into account.

Small Size:
With a few exceptions, such as some law firms and accounting firms, professional organizations are
relatively small and operate at a single location. Senior management in such organizations can personally
observe what is going on and personally motivate employees. Thus, there is less need for a sophisticated
management control system, with profit centers and formal performance reports. Nevertheless, even a small
organization needs a budget, a regular comparison of performance against budget, and a way of relating
compensation to performance.

Marketing:
In a manufacturing company there is a clear dividing line between marketing activities and production
activities; only senior management is concerned with both. Such a clean separation does not exist in most
professional organizations. In some, such as law, medicine, and accounting, the profession's ethical code limits
the amount and character of overt marketing efforts by professionals (although these restrictions have been
relaxed in recent years). Marketing is an essential activity in almost all organizations, however. If it can't be
conducted openly, it takes the form of personal contacts, speeches, articles, conversations on the golf course, and
so on. These marketing activities are conducted by professionals, usually by professionals who spend much of
their time in production work-that is, working for clients.

In this situation, it is difficult to assign appropriate credit to the person responsible for "selling" a new customer.
In a consulting firm, for example, a new engagement may result from a conversation between a member of the
firm and an acquaintance in a company, or from the reputation of one of the firm's professionals as an outgrowth
of speeches or articles. Moreover, the professional who is responsible for obtaining the engagement may not be
personally involved in carrying it out. Until fairly recently, these marketing contributions were rewarded
subjectively- that is, they were taken into account in promotion and compensation decisions. Some organizations
now give explicit credit, perhaps as a percentage of the project's revenue, if the person who "sold" the project can
be identified.

Every SBU is a profit center but every profit center is not a SBU? What are the conditions that should
be fulfill for an organization unit to be converted into a profit center? What are the different ways to
measure the performance of profit center? Discuss their relevant merits and demerits.

Conditions for an organization to be converted into a profit centre:


Many management decisions involve proposals to increase expenses with the expectation of an even greater
increase in sales revenue. Such decisions are said to involve expense/revenue trade-offs. Additional advertising
expense is an example. Before it is safe to delegate such a trade-off decision to a lower-level manager, two
conditions should exist.

• The manager should have access to the relevant information needed for making such a decision.
• There should be some way to measure the effectiveness of the trade-offs the manager has made.

55
A major step in creating profit centers is to determine the lowest point in an organization where these two
conditions prevail. All responsibility centers fit into a continuum ranging from those that clearly should be profit
centers to those that clearly should not. Management must decide whether the advantages of giving profit
responsibility offset the disadvantages, which are discussed below. As with all management control system
design choices, there is no clear line of demarcation.

Ways to Measure Performance:


There are two types of profitability measurements used in evaluating a profit center, just as there are in
evaluating an organization as a whole. First, there is the measure of management performance, which focuses on
how well the manager is doing. This measure is used for planning, coordinating, and controlling the profit
center's day-to-day activities and as a device for providing the proper motivation for its manager. Second, there
is the measure of economic performance, which focuses on how well the profit center is doing as an economic
entity. The messages conveyed by these two measures may be quite different from each other. For example, the
management performance report for a branch store may show that the store's manager is doing an excellent job
under the circumstances, while the economic performance report may indicate that because of economic and
competitive conditions in its area the store is a losing proposition and should be closed. .

The necessary information for both purposes usually cannot be obtained from a single set of data. Because the
management report is used frequently, while the economic report is prepared only on those occasions when
economic decisions must be made, considerations relating to management performance measurement have first
priority in systems design-that is, the system should be designed to measure management performance routinely,
with economic information being derived from these performance reports as well as from other sources.

Types of Profitability Measures


A profit center's economic performance is always measured by net income (i.e., the income remaining after
all costs, including a fair share of the corporate overhead, have been allocated to the profit center). The
performance of the profit center manager, however, may be evaluated by five different measures of profitability:
(1) contribution margin, (2) direct profit, (3) controllable profit, (4) income before income taxes, or (5) net
income

(1) Contribution Margin:


Contribution margin reflects the spread between revenue and variable expenses. The principal argument in favor
of using it to measure the performance of profit center managers is that since fixed expenses are beyond their
control, managers should focus their attention on maximizing contribution. The problem with this argument is
that its premises are inaccurate; in fact, almost all fixed expenses are at least partially controllable by the
manager, and some are entirely controllable. Many expense items are discretionary; that is, they can be changed
at the discretion of the profit center manager. Presumably, senior management wants the profit center to keep
these discretionary expenses in line with amounts agreed on in the budget formulation process. A focus on the
contribution margin tends to direct attention away from this responsibility. Further, even if an expense, such as
administrative salaries, cannot be changed in the short run, the profit center manager is still responsible for
controlling employees' efficiency and productivity.

(2) Direct Profit:


This measure reflects a profit center's contribution to the general overhead and profit of the corporation. It
incorporates all expenses either incurred by or directly traceable to the profit center, regardless of whether or not
these items are within the profit center manager's control. Expenses incurred at headquarters, however, are not
included in this calculation. A weakness of the direct profit measure is that it does not recognize the motivational
benefit of charging headquarters costs.

(3) Controllable Profit:


Headquarters expenses can be divided into two categories: controllable and non controllable. The former
category includes expenses that are controllable, at least to a degree, by the business unit manager-information
technology services, for example. If these costs are included in the measurement system, profit will be what
remains after the deduction of all expenses that may be influenced by the profit center manager. A major
disadvantage of this measure is that because it excludes non controllable headquarters expenses it cannot be
directly compared with either published data or trade association data reporting the profits of other companies in
the industry.

(4) Income before Taxes:


In this measure, all corporate overhead is allocated to profit centers based on the relative amount of expense each
profit center incurs. There are two arguments against such allocations. First, since the costs incurred by corporate
staff departments such as finance, accounting, and human resource management are not controllable by profit
center managers, these managers should not be held accountable for them. Second, it may be difficult to allocate
56
corporate staff services in a manner that would properly reflect the amount of costs incurred by each profit
center.

There are, however, three arguments in favor of incorporating a portion of corporate overhead into the profit
centers' performance reports. First, corporate service units have a tendency to increase their power base and to
enhance their own excellence without regard to their effect on the company as a whole. Allocating corporate
overhead costs to profit centers increases the likelihood that profit center manager§ will question these costs,
thus serving to keep head office spending in check. (Some companies have actually been known to sell their
corporate jets because of complaints from profit center managers about the cost of these expensive items.)

Second, the performance of each profit center will become more realistic and more readily comparable to the
performance of competitors who pay for similar services. Finally, when managers know that their respective
centers will not show a profit unless all-costs, including the allocated share of corporate overhead, are recovered,
they are motivated to make optimum long-term marketing decisions as to pricing, product mix, and so forth, that
will ultimately benefit (and even ensure the viability of) the company as a whole.

If profit centers are to be charged for a portion of corporate overhead, this item should be calculated on the basis
of budgeted, rather than actual, costs, in which case the "budget" and "actual" columns in the profit center's
performance report will show identical amounts for this particular item. This ensures that profit center managers
will not complain about either the arbitrariness of the allocation or their lack of control over these costs, since
their performance reports will show no variance in the overhead allocation. Instead, such variances would appear
in the reports of the responsibility center that actually incurred these costs. .

(5) Net Income:


Here, companies measure the performance of domestic profit centers according to the bottom line, the amount of
net income after income tax. There are two principal arguments against using this measure:

(1) after tax income is often a constant percentage of the pretax income, in which case there would be no
advantage in incorporating income taxes

(2) since many of the decisions that affect income taxes are made at headquarters, it is not appropriate to judge
profit center managers on the consequences of these decisions.

There are situations, however, in which the effective income tax rate does vary among profit centers. For
example, foreign subsidiaries or business units with foreign operations may have different effective income tax
rates. In other cases, profit centers may influence income taxes through their installment credit policies, their
decisions on acquiring or disposing of equipment, and their use of other generally accepted accounting
procedures to distinguish gross income from taxable income. In these situations, it may be desirable to allocate
income tax expenses
to profit centers not only to measure their economic profitability but also to motivate managers to minimize tax
liability.

Merits:

• The quality of decisions may improve because they are being made by managers closest to the point of decision.
• The speed of operating decisions may be increased since they do not have to be referred to corporate
headquarters. . Headquarters management, relieved of day-to-day decision making, can concentrate on broader
issues.
• Managers, subject to fewer corporate restraints, are freer to use their imagination and initiative.
• Because profit centers are similar to independent companies, they provide an excellent training ground for
general management. Their managers gain experience in managing all functional areas, and upper management
gains the opportunity to evaluate their potential for higher-level jobs.
• Profit consciousness is enhanced since managers who are responsible' for profits will constantly seek ways to
increase them. (A manager responsible for marketing activities, for example, will tend to authorize promotion
expenditures that increase sales, whereas a manager responsible for profits will be motivated to make promotion
expenditures that increase profits.).
• Profit centers provide top management with ready-made information on the profitability of the company's
individual components. . Because their output is so readily measured, profit centers are particularly responsive to
pressures to improve their competitive performance.

57
Demerits:

• Decentralized decision making will force top management to rely more on management control reports than on
personal knowledge of an operation, entailing some loss of control.
• If headquarters management is more capable or better informed than the average profit center manager, the
quality of decisions made at the unit level may be reduced.
• Friction may increase because of arguments over the appropriate transfer price, the assignment of common costs,
and the credit for revenues that were formerly generated jointly by two or more business units working together.
• Organization units that once cooperated as functional units may now be in competition with one another. An
increase in profits for one manager may mean a decrease for another. In such situations, a manager may fail to
refer sales leads to another business unit better qualified to pursue them; may hoard personnel or equipment that,
from the overall company standpoint, would be better off used in another unit; or may make production
decisions that have undesirable cost consequences for other units.
• Divisionalization may impose additional costs because of the additional management, staff personnel, and record
keeping required, and may lead to task redundancies at each profit center.

What are different types of Strategic Missions at SBU level?


How do these missions affect Strategic Planning process and Budgeting at SBU Level?

Different Types of Strategic Missions:

Business Unit Mission:

In a diversified firm one of the important tasks of senior management is resource deployment, that is, make
decisions regarding the use of the cash generated from some business units to finance growth in other business
units. Several planning models have been developed to help corporate level managers of diversified firms to
effectively allocate resources. These models suggest that a firm has business units in several categories,
identified by their mission; the appropriate strategies for each category differ. Together, the several units make
up a portfolio, the components of which differ as to their risk/reward characteristics just as the components of an
investment portfolio differ.

Both the corporate 'office and the business unit general manager are involved in identifying the missions of
individual business units. Of the many planning models, two of the most widely used are Boston Consulting
Group's two-by-two growth-share matrix and General Electric Company/McKinsey & Company's three-by-three
industry attractiveness-business strength matrix. While these models differ in the methodologies they use to
develop the most appropriate missions for the various business units, they have the same set of missions from
which to choose: build, hold, harvest, and divest.

Build:
This mission implies an objective of increased market share, even at the expense of short-term earnings and cash
flow (e.g., Merck's bio-technology, Black and Decker's handheld electric tools).

Hold:
This strategic mission is geared to the protection of the business unit's market share and competitive position
(e.g.: IBM's mainframe computers).

Harvest:
This mission has the objective of maximizing short-term earnings and cash flow, even at the expense of market
share (e.g., American Brands' tobacco products, General Electric's and Sylvania's light bulbs)

Divest:
This mission indicates a decision to withdraw from the business either through a process of slow liquidation or
outright sale. While the planning models can aid in the formulation of missions, they are not cook books. A
business unit's position on a planning grid should not be the sole basis for deciding its mission.

Business Unit Competitive Advantage:

58
Every business unit should develop a competitive advantage in order to accomplish its mission. Three
interrelated questions have to be considered in developing the business unit's competitive= advantage. First, what
is the structure of the industry in which the business unit operates? Second, how should the business unit exploit
the industry's structure? Third, what will be the basis of the business unit's competitive advantage?

Industry Analysis:
Research has highlighted the important role industry conditions play in the performance of individual firms.
Studies have shown that average industry profitability is, by far, the most significant predictor of firm
performance. According to Porter, the structure of an industry should be analyzed in terms of the collective
strength of five competitive forces.

1. The intensity of rivalry among existing competitors. Factors affecting direct rivalry are industry growth,
product differentiability, number and diversity of competitors, level of fixed costs, intermittent overcapacity, and
exit barriers.

2. The bargaining power of customers. Factors affecting buyer power are number of buyers, buyer's switching
costs, buyer's ability to integrate backward, impact of the business unit's product on buyer's total costs, impact of
the business unit's product on buyer's product quality/ performance, and significance of the business unit's
volume to buyers.

3. The bargaining power of suppliers. Factors affecting supplier power are number of suppliers, supplier's ability
to integrate forward, presence of substitute inputs, and importance of the business unit's volume to suppliers.

4. Threat from substitutes. Factors affecting substitute threat are relative price/performance of substitutes, buyer's
switching costs, and buyer's propensity to substitute.

5. The threat of new entry. Factors affecting entry barriers are capital requirements, access to distribution
channels, economies of scale, product differentiation, technological complexity of product or process, expected
retaliation from existing firms, and government policy.

• We make three observations with regard to the industry analysis:

1. The more powerful the five forces are, the less profitable an industry is likely to be. In industries where
average profitability is high (such as soft drinks and pharmaceuticals), the five forces are weak (e.g., in the soft
drink industry, entry barriers are high). In industries where the average profitability is low (such as steel and
coal), the five forces are strong (e.g., in the steel industry, threat from substitutes is high).

2. Depending on the relative strength of the five forces, the key strategic issues facing the business unit will
differ from one industry to another.

3. Understanding the nature of each force helps the firm to formulate effective strategies. Supplier selection (a
strategic issue) is aided by the analysis of the relative power of several supplier groups; the business unit should
link with the supplier group for which it has the best competitive advantage. Similarly, analyzing the relative
bargaining power of several buyer groups will facilitate selection of target customer segments.

Generic Competitive Advantage:


The five-force analysis is the starting point for developing a competitive advantage since it helps to identify the
opportunities and threats in the external environment. With this understanding, Porter claims that the business
unit has two generic ways of responding to the opportunities in the external environment and developing a
sustainable competitive advantage: low cost and differentiation.

• Low Cost:
Cost leadership can be achieved through such approaches as economies of scale in production; experience
curve effects, tight cost control, and cost minimization (in such areas as research and development, service, sales
force, or advertising). Some firms following this strategy include Charles Schwab in discount brokerage, Wal-
Mart in discount retailing, Texas Instruments in consumer electronics, Emerson Electric in electric motors,
Hyundai in automobiles, Dell in computers, Black and Decker in machine tools, Nucor in steel, Lincoln Electric
in arc welding equipment, and BIC in pens.

• Differentiation:
The primary focus of this strategy is to differentiate the product offering of the business unit, creating
something that is perceived by customers as being unique. Approaches to product differentiation include brand

59
loyalty (Coca-Cola and Pepsi Cola in soft drinks), superior customer service (Nordstrom in retailing), dealer
network (Caterpillar Tractors in construction equipment), product design and product features (Hewlett-Packard
in electronics), and technology (Cisco in communications infrastructure). Other examples of firms following a
differentiation strategy include BMW in automobiles; Stouffer's in frozen foods, Neiman-Marcus in retailing,
Mont Blanc in pens, and Rolex in wristwatches.

Value Chain Analysis:


Business units can develop competitive advantage based on low cost, differentiation, or both. The most attractive
competitive position is to achieve cost-cum-differentiation.

Pritam Engineering manufacturers (MCS-2005) Numerical

Pritam Engineering manufacturing variety of metal product at many factories. Currently. It is


experiencing crisis, Management has, therefore, decided to detailed expense control system including
responsibility budgets for overhead expense items at each factory. From historical data, Controller developed a
standard for each overhead expense item (relating expense to volume of activity). Summarized expenses for
November,2005 given to concerned Production Supervisor for comments is tabulated. [All figures are in Rs.
000]

Item Standard at nominal volume Budgeted at actual volume actual

Management Supervision 720 720 582


Indirect labour 12706 11322 12552
Idle time 420 361 711
Materials, Tools 3600 3096 3114
Maintenance, scrap 14840 13909 17329
Allocated expenses 21040 21040 21218
Total per ton (Rs.) 2133.04 2103.39 2413.3

(A) Explain with justification which of the two (1) or (2) is more meaningful for expense control.
(B) Can the supervisor be held responsible for all overhead expenses included? Why/why not?

Solution

(A) There is two general types of expense centers: engineered and discretionary. This label relate to two types of
cost. Engineered costs are those for which the “right” or “proper” amount can be estimated with reasonable
reliability for example, a factory’s costs for direct labor, direct material, components, supplies, and utilities.
Discretionary costs (also called managed costs) are those for which not such engineered estimate is feasible.
In discretionary expense centers, the costs incurred depend on managements judgment as to the appropriate
amount under the circumstances.

Engineered expense centers


Engineered expense centers are usually found a manufacturing operations. Warehousing, distribution,
trucking, and similar units within the marketing organization may also be engineered expense centers, as may
certain responsibility centers within administrative and support department for instance, accounts receivable,
accounts payable, and payroll sections in the controller department; personnel records and the cafeteria in the
human resources department; shareholder records in the corporate secretary department; and the company
motor pool. Such units perform repetitive tasks for which standard costs can be developed. These engineered
expense centers are usually located within departments that are discretionary expense centers.
In an engineered expense center, output multiplied by the standard cost of each unit produced measures what
the finished product should have cost. The difference between the theoretical and the actual cost represents the
efficiency of the expense center being measure.

60
We emphasize that engineered expense centers have other important tasks not measured by cost alone; their
supervisors are responsible for the quality of the products and volume of production as well as for efficiency.
Therefore, the type and level of production are prescribed, and specific quality standards are set. So that
manufacturing costs are not minimized at the expense of quality. Moreover, managers of engineered expense
centers may be responsible for activities such as training and employee development that are not related to
current production; their performance reviews should include an appraisal of how well they carry out these
responsibilities.

There are few, if any, responsibility centers in which all cost items are engineered. Even in highly automated
production departments, the use of indirect labor and various services can vary with management’s discretion.
Thus the term engineered expense center refers to responsibility centers in which engineered costs predominate.
But it does not imply that valid engineered estimates can be made for each and every cost item.

Discretionary expense centers


Discretionary expense centers include administrative and support units (e.g. accounting, legal, industrial
relations, public relations, human resources), research and development operations, and most marketing
activities. The output of these centers cannot be measured in monetary terms.

The term discretionary does into imply that managements judgment as to optimum cost is capricious or
haphazard. Rather it reflects management’s decisions regarding certain policies: whether to match or exceed the
marketing efforts of competitors; the level of services the company should provide to its customers; and the
appropriate amounts to spend for R&D, financial planning, public relations, and a host of other activities.
One company may have a small headquarters staff, while another company of similar size and in the same
industry may have a staff 10 times as large. The senior managers of each company may each be convinced that
their respective decisions on staff size are correct, but there is no objective way to judge which (if either) is right;
both decisions may be equally good under the circumstances, with the differences’ in size reflecting other
underlying deference’s in the two companies.

As far as above stated over heads are concern, we can easily estimate “proper” or “right” amount with
responsible reliability. There for standard (1) is more meaningful for expenses control.

(B) A responsibility center is an organization unit that is headed by a manager who is responsible for its
activities. In a sense, a company is a collection of responsibility centers, each of which is represented by a
box on the organization chart. These responsibility centers form a hierarchy. At the lowest level are the
centers of the sections, work shift, and other small organization units. Departments or business units
comprising several of these smaller units are higher in the hierarchy. From the standpoint of senior
management and and the board of directors, the entire company is a responsibility center, though the term
is usually used to refer to units within the company and there for Supervisor is responsible for the uses of
the Above stated Resources (over heads) like Indirect labor, idle time, Materials, tools, maintenance, scrape
and Management supervision by proper supervising supervisor can control the listed overhead expenses.

What is a responsibility centre? List and explain different types of Responsibility Centers.

Responsibility centers:

A responsibility center is an organization unit that is headed by a manager who is responsible for its activities. In
a sense, a company is a collection of responsibility centers. Each of which is represented by box on the on the
organization are responsibility centers for section work shifts or other small organization units. At a higher level
are departments or business units that consist of several of these smaller units plus staff and management people
these larger units are also responsibility center. And from the stand point of senior management and the board of
directors, the whole company is responsibility center although the term is usually used to refer to unit within the
company.

61
Nature of responsibility centers

A responsibility center exist one or more purpose are its objectives. The company as a whole has goals, and
senior management has decided on a set of strategies to accomplish these goals. The objectives of responsibility
centers are to help implement these strategies. Because the organization is the sum of its responsibility centers, if
the strategies are sound and if each responsibility center, if the strategies are sound and if each responsibility
center meets its objectives the whole organization should achieve its goals. A responsibility center uses inputs,
and a variety of services. Its work with these resources and it usually require working capital, equipment, and
other asset to do this work. As a result of this work the responsibility center produces output which is classified
either as goods if they are tangible or as services if they are intangible. Every responsibility center has output
that is it does something. In a production plant, the outputs are goods. In staff units, such as human resources,
transportation, engineering, accounting, and administration, the output s are services. For many responsibility
centers, especially staff units, outputs are difficult to measure; nevertheless, they exist. The products produced by
a responsibility center or to the outside marketplace. In the first case, the product are inputs to the other
responsibility center in the latter case, they are output s of the whole organization.

Types of Responsibility Centers

a) Cost Center
Cost centers are divisions that add to the cost of the organization, but only indirectly add to the profit of the
company. Typical examples include Research and Development, Marketing and Customer service. Companies
may choose to classify business units as cost centers, profit centers, or investment centers. There are some
significant advantages to classifying simple, straightforward divisions as cost centers, since cost is easy to
measure. However, cost centers create incentives for managers to underfund their units in order to benefit
themselves, and this underfunding may result in adverse consequences for the company as a whole (reduced
sales because of bad customer service experiences, for example). Because the cost centre has a negative impact
on profit (at least on the surface) it is a likely target for rollbacks and layoffs when budgets are cut. Operational
decisions in a contact centre, for example, are typically driven by cost considerations. Financial investments in
new equipment, technology and staff are often difficult to justify to management because indirect profitability is
hard to translate to bottom-line figures. Business metrics are sometimes employed to quantify the benefits of a
cost centre and relate costs and benefits to those of the organization as a whole. In a contact centre, for example,
metrics such as average handle time, service level and cost per call are used in conjunction with other
calculations to justify current or improved funding.

b) Profit Center

A responsibility centre is called a profit centre when the manager is held responsible for both costs (inputs)
and revenues (outputs) and thus for profit. Despite the name, a profit centre can exist in nonprofits organizations
(though it might not be referred to as such) when a responsibility centre receives revenues for its services. A
profit centre is a big segment of activity for which both revenues and costs are accumulated: A centre, whose

62
performance is measured in terms of both - the expense it incurs and revenue it earns, is termed as a profit centre.
The output of a responsibility centre may either be meant for internal consumption or for outside customers. In
the latter case, the revenue is realized when the sales are made. That is, when the output is meant for outsiders,
then the revenue will be measured from the price charged from customers. If the output is meant for other
responsibility centre, then management takes a decision whether to treat the centre as profit centre or not. In fact,
any responsibility centre can be turned into a profit centre by determining a selling price for its outputs. For
instance, in case of a process industry, the output of one process may be transferred to another process at a profit
by taking into account the market price. Such transfers will give some profit to that responsibility centre.
Although such transfers do not increase the Company’s assets, they help in management control process.

c) Investment Centre

An investment centre goes a step further than a profit centre does. Its success is measured not only by its
income but also by relating that income to its invested capital, as in a ratio of income to the value of the capital
employed. In practice, the term investment centre is not widely used. Instead, the term profit centre is used
indiscriminately to describe centers that are always assigned responsibility for revenues and expenses, but may
or may not be assigned responsibility for the capital investment. It is defined as a responsibility centre in which
inputs are measured in terms of cost / expenses and outputs are measured in terms of revenues and in which
assets employed are also measured. A responsibility centre is called an investment centre, when its manager is
responsible for costs and revenues as well as for the investment in assets used by his centre. He is responsible for
maintaining a satisfactory return on investment i.e. asset employed in his responsibility centre. The investment
centre manager has control over revenues, expenses and the amounts invested in the centre’s assets. The
manager of an investment centre is required to earn a satisfactory return. Thus, return on investment (ROI) is
used as the performance evaluation criterion in an investment centre. He also formulates the credit policy, which
has a direct influence on debt collection, and the inventory policy, which determines the investment in inventory.
The Vice President (Investments) of a mutual funds company may be in charge of an Investment Centre. In the
Investment Centre, the manager in charge is held responsible for the proper utilization of assets. He is expected
to earn a satisfactory return on the assets employed in his responsibility centre. Measurement of assets employed
poses many problems. It becomes difficult to determine the amount of assets employed in a particular
responsibility centre. Some of the assets are in the physical possession of the responsibility centre while for some
assets it may depend upon other responsibility centers or the Head Office of the company. This is particularly
true of cash or heavy plant and equipment. Whether such assets should be included in the figure of assets
employed of the responsibility centre and if included, at how much value, is a difficult question. On account of
these difficulties, investment centers are generally used only for relatively large units, which have independent
divisions, both manufacturing and marketing, for their individual products.

Explain the process of evaluation of Responsibility Center from one stage to another with the help of
illustration-cum-experiences of the corporate.

63
Process of evaluation of Responsibility Center.

1. The organization is divided into various responsibility centers. Each responsibility centre is put under
the charge of a responsibility manager.

2. The targets or budgets of each responsibility centre are set in consultation with the manager of
responsibility centre, so that he may be able to give full information about his department. The manager
of responsibility centre should know as what is expected of him - each centre should have a clear set of
goals. The responsibility and authority of each centre should be well defined.

3. Managers are charged with the items and responsibility, over which they can exercise a significant
degree of direct control.

4. Goals defined for each area of responsibility should be attainable with efficient and effective
performance.

5. The actual performance is communicated to the managers concerned. If it falls short of the standards,
the variances are conveyed to the top management. The names of persons responsible for the variances
are also conveyed so that responsibility may be fixed.

The purpose of all these steps is to assign responsibility to different individuals so that their performance is
improved and costs are controlled. The personal factor in Responsibility Accounting is most important. The
management may prepare the best plan or the budget and put up before its staff, but its success depends upon
the initiative and the will of the workers to execute it

Example of Responsibility Center

The Sarva Shiksha Abhiyan emphasizes quality improvement in elementary education for which it
deems necessary that resource groups and responsibility centers from national to sub-district levels are identified.
These groups would oversee the policy, planning, implementation and monitoring of all quality related
interventions. Their major role would be to advise and assist at various levels in curriculum development,
pedagogical improvement, teacher education/training and activities related to classroom transaction. In order to
facilitate a decentralized mode of education, these groups would need to be constituted at various operational
levels, namely - national, state, district and sub district. The following could be involved in the groups:
National level - NCERT, NIEPA, Universities, NGOs, experts and eminent educationists.
State level - SCERT, SIEMAT, Universities, IASEs/CTEs, NGOs, experts and eminent educationists.
District level - DIETs, representatives from DPEP District Resource Group, higher educational institutions,
innovative teachers from the districts, NGOs.
Sub-district - BRC/BEO, representatives from CRCs, innovative teachers.

Briefly define Discretionary Expense Center, Engineered Expense Center, Profit Centre and Investment
Centre? How is budget prepared in Discretionary Expenses Centre?

Engineered expense centers:

Engineered expense center have the following characteristics:

- Their inputs can be measured in monetary terms.


- Their output can be measured in physical terms.
- The optimal dollar amount of input required to produce one unit of output can be established.

64
Engineered expense center usually are found in manufacturing operations. Warehousing, distribution, trucking
and similar units in the marketing organization also may be engineered expense center and so many certain
responsibility center within administrative and support department. Examples are accounts receivable account
payable and payroll section in the controller department personnel record and cafeteria in the human resource
department shareholder record in the corporate secretary department and the company motor pool. Such units
perform repetitive task for which standard cost can be developed. In an engineered expense center the output
multiplied by the standard cost of each unit produced represents what the finished product should have cost.
When this cost is compared to actual costs, the difference between the two represents the efficiency of the
organization unit being measured. We emphasize that engineered expense centers have other important tasks not
measured by cast alone. The effectiveness of this aspect of performance should be controlled. For example
expenses center supervisor are responsible for the quality of good and for the volume of production in addition to
their responsibility for cost efficiency. Therefore the type and amount of production is prescribed and specific
quality standards are set so that manufacturing costs are not minimized at the expense of quality. Moreover
manager of engineered expense center may be responsible for activities such a training that are not related to
current production judgment about their performance should include an appraisal of how well they carry out
these responsibilities. There are few if any responsibility center in which all cost items are engineered. Even in
highly automated production department the amount of indirect labor and of various services used can vary with
management discretion. Thus, the term engineered costs center refers to responsibility center in which
engineered cost predominate but in does not imply that valid engineering estimates can be made for each and
every cost item.

Discretionary expense center:

The output of discretionary expenses center cannot be measured in monitory terms. They include administration
and support units research and development organization and most marketing activities. The term discretionary
does not mean that management judgment is capricious or haphazard. Management has decided on certain
policies that should govern the operation of the company. Whether to match exceed or spend less than the
marketing effort of its competitor; the level of service that the company provides to the customer. The
appropriate amount of spending for R & D, financial planning public relation and many other activities. One
company may have a small headquarter staff another company of similar size and in the same industry may have
a staff that is 10 times as large. the management of both companies may be concerned that they made the correct
decision on staff size but there is no objective way judging which decision was actually better manager are hired
and paid to make such decision. After such a drastic change the level of discretionary expenses generally has a
similar pattern from one year to the next. The difference between budgeted and actual expense is not a measure
of efficiency in a discretionary expense center it is simply the difference between the budgeted input and the
actual input. It in no way measures the value of the output. if actual expense do not exceed the budget amount,
the manager has ‘lived within the budget ‘ however ,because by definition the budget does not purport to
measure the optimum amount of spending we cannot say that living within the budgeted is efficient performance.

Budget Preparation.

The decision that management make about a discretionary expense budget are different from the
decisions that it makes about the budget for an engineered expense center. For the latter management decides
whether the proposed operating budget represent the cost of performing task efficiently for the coming period.
management is not so much concerned with the magnitude of the task because this is largely determined by the
actions of other responsibility centers, such as the marketing departments ability to generate sales. In formulating
the budget for a discretionary expense center, however management principal task is to decide on the magnitude
of the job that should be done. These tasks can be divided generally into two types continuing and special.
Continuing task are those that continue from year to year for example financial statement preparation by the
controller’s office. Special tasks are one shot project for example developing and installing a profit budgeting
system in a newly acquired division. The technique management by objective is often used in preparing the
budget for a discretionary expense center. Management by objective is a formal process in which a budget
purposes to accomplish specific tasks and state a mean for measuring whether these tasks have been
accomplished. There are two different approach to planning for the discretionary expense center increment
budgeting and zero based review.

Describe inherent difficulties creation of profit centres may cause and advantages possible?
Under which situation creation of profit centre is not advisable.

65
Under which situation creation of profit centre is not advisable

Decentralized decision making will force top management to rely more on management control reports
than on personal knowledge of an operation, entailing some loss of control. If headquarters management is mere
capable or better informed than the average profit center manager, the quality of decisions made at the unit level
way be reduced. Friction may increase because of arguments over the appropriate transfer price, the assignment
of common costs, and the credit for revenues that were formerly generated jointly by two or more business units
working together.

Organization units that once cooperated as functional units may now be in competition with one
another. An increase in profits for one manager may mean a decrease for another. In such situation a manager
may fail to refer sales leads to another business unit better qualified to pursue them; may hoard personnel or
equipment that, from the overall company’s, standpoint, would be better off used in another unit; or may make
production decisions that have undesirable cost consequences for other units.

Divisionalization may impose additional costs because of the additional management, staff personnel,
and record keeping required, and may lead to task redundancies at each profit center.Competent general
managers may not exist in a functional organization because there may not have been sufficient opportunities for
them to develop general management competence.

There may be too much emphasis on short-run profitability at the expense of long-run profitability. In
the desire to report high current profits, the profit center manager may skimp on R&D, training programs, or
maintenance. This tendency is especially prevalent when the turnover of profit center managers is relatively
high. In these circumstances, managers may have good reason to believe that their actions may not affect
profitability until after they have moved to other jobs. There is no completely satisfactory system for ensuring
that optimizing the profits of each individual profit center will optimize the profits of the company as a whole.

What are the challenges faced in pricing corporate services provided to Business Units operating
as “profit centers?”

Business Units as Profit Centers


Most business units are created as profit centers since managers in charge of such units typically control product
development, manufacturing, and marketing resources. These managers are in a position to influence revenues
and costs and as such can be held accountable for the "bottom line." However, as pointed out in the next section,
a business unit manager's authority may be constrained in various ways, which ought to be reflected in a profit
center's design and operation.

Constraints on Business Unit Authority


To realize fully the benefits of the profit center concept, the business unit manager would have to be as
autonomous as the president of an independent company. As a practical matter, however, such autonomy is not
feasible. If a company were divided into completely independent units, the organization would lose the
advantages of size and synergy. Furthermore in delegating to business unit management all the authority that the
board of directors has given to the CEO, senior management would be abdicating its own responsibility.
Consequently, business unit structures represent trade-offs between business unit autonomy and corporate
constraints. The effectiveness of a business unit organization is largely dependent on how well these trade-offs
are made.

Constraints from Other Business Units.


One of the main problems occurs when business units must deal with one another. It is useful to think of
managing a profit center in terms of control over three types of decisions:
(1) The product decision (what goods or services to make and sell),
(2) The marketing decision (how, where, and for how much are these goods or services to be sold?), and
(3) The procurement or sourcing decision (how to obtain or manufacture the goods or services). If a business
unit manager controls all three activities, there is usually no difficulty in assigning profit responsibility and
measuring performance. In general, the greater the degree of integration within a company,
the more difficult it becomes to assign responsibility to a single profit center for all three activities in a given
product line; that is, if the production, procurement, and marketing decisions for a single product line are split

66
among two or more business units, separating the contribution of each business unit to the overall success of the
product line may be difficult.

Constraints from Corporate Management

The constraints imposed by corporate management can be grouped into three types:
(1) Those resulting from strategic considerations,
(2) Those resulting because uniformity is required, and
(3) Those resulting from the economies of centralization.

Most companies retain certain decisions, especially financial decisions, at the corporate level, at least for
domestic activities. Consequently, one of the major constraints on business units results from corporate control
over new investments. Business units must compete with one another for a share of the available funds. Thus, a
business unit could find its expansion plans thwarted because another unit has convinced senior management that
it has a more

Attractive program. Corporate management .also imposes other constraints. Each business unit has a "charter"
that specifies the marketing and/or production activities that it is permitted to undertake, and it must refrain from
operating beyond its charter, even though it sees profit opportunities in doing so. Also, the maintenance of the
proper corporate image may require constraints on the quality of products or on public relations activities.

Companies impose some constraints on business units because of the necessity for Uniformity. One-constraint is
that business Units must conform to corporate accounting and MCS This constraint is especially troublesome for
units that have been acquired from another company and that have been accustomed to using different systems.

Zero Based Budgeting

Zero-based budgeting is a technique of planning and decision-making which reverses the working
process of traditional budgeting. In traditional incremental budgeting, departmental managers justify only
increases over the previous year budget and what has been already spent is automatically sanctioned. No
reference is made to the previous level of expenditure. By contrast, in zero-based budgeting, every department
function is reviewed comprehensively and all expenditures must be approved, rather than only increases.[1] Zero-
based budgeting requires the budget request be justified in complete detail by each division manager starting
from the zero-base. The zero-base is indifferent to whether the total budget is increasing or decreasing.
The term "zero-based budgeting" is sometimes used in personal finance to describe the practice of
budgeting every dollar of income received, and then adjusting some part of the budget downward for every other
part that needs to be adjusted upward. It would be more technically correct to refer to this practice as "active-
balanced budgeting".
Advantages of Zero-Based Budgeting:
1. Efficient allocation of resources, as it is based on needs and benefits.
2. Drives managers to find cost effective ways to improve operations.
3. Detects inflated budgets.
4. Municipal planning departments are exempt from this budgeting practice.
5. Useful for service departments where the output is difficult to identify.
6. Increases staff motivation by providing greater initiative and responsibility in decision-making.
7. Increases communication and coordination within the organization.
8. Identifies and eliminates wasteful and obsolete operations.
9. Identifies opportunities for outsourcing.
10. Forces cost centers to identify their mission and their relationship to overall goals.
Disadvantages of Zero-Based Budgeting:
1. Difficult to define decision units and decision packages, as it is time-consuming and exhaustive.

67
2. Forced to justify every detail related to expenditure. The R&D department is threatened whereas the
production department benefits.
3. Necessary to train managers. Zero-based budgeting must be clearly understood by managers at various
levels to be successfully implemented. Difficult to administer and communicate the budgeting because
more managers are involved in the process.
4. In a large organization, the volume of forms may be so large that no one person could read it all.
Compressing the information down to a usable size might remove critically important details.
5. Honesty of the managers must be reliable and uniform. Any manager that exaggerates skews the results

Internal Control

Internal control is defined as a process affected by an organization's structure, work and authority
flows, people and management information systems, designed to help the organization accomplish specific
goals or objectives.[1] It is a means by which an organization's resources are directed, monitored, and measured.
It plays an important role in preventing and detecting fraud and protecting the organization's resources, both
physical (e.g., machinery and property) and intangible (e.g., reputation or intellectual property such as
trademarks). At the organizational level, internal control objectives relate to the reliability of financial
reporting, timely feedback on the achievement of operational or strategic goals, and compliance with laws and
regulations. At the specific transaction level, internal control refers to the actions taken to achieve a specific
objective (e.g., how to ensure the organization's payments to third parties are for valid services rendered.)
Internal control procedures reduce process variation, leading to more predictable outcomes

Describing Internal Controls:


Internal controls may be described in terms of: a) the objective they pertain to; and b) the nature of the control
activity itself.
Objective categorization
Internal control activities are designed to provide reasonable assurance that particular objectives are achieved, or
related progress understood. The specific target used to determine whether a control is operating effectively is
called the control objective. Control objectives fall under several detailed categories; in financial auditing, they
relate to particular financial statement assertions,[5] but broader frameworks are helpful to also capture
operational and compliance aspects:
1. Existence (Validity): Only valid or authorized transactions are processed (i.e., no invalid transactions)
2. Occurrence (Cutoff): Transactions occurred during the correct period or were processed timely.
3. Completeness: All transactions are processed that should be (i.e., no omissions)
4. Valuation: Transactions are calculated using an appropriate methodology or are computationally
accurate.
5. Rights & Obligations: Assets represent the rights of the company, and liabilities its obligations, as of a
given date.
6. Presentation & Disclosure (Classification): Components of financial statements (or other reporting) are
properly classified (by type or account) and described.
7. Reasonableness-transactions or results appear reasonable relative to other data or trends.
Activity categorization
Control activities may also be described by the type or nature of activity. These include (but are not limited to):
• Segregation of duties - separating authorization, custody, and record keeping roles to limit risk of fraud
or error by one person.
• Authorization of transactions - review of particular transactions by an appropriate person.
• Retention of records - maintaining documentation to substantiate transactions.
• Supervision or monitoring of operations - observation or review of ongoing operational activity.
• Physical safeguards - usage of cameras, locks, physical barriers, etc. to protect property.
• Analysis of results, periodic and regular operational reviews, metrics, and other key performance
indicators (KPIs).
• IT Security - usage of passwords, access logs, etc. to ensure access restricted to authorized personnel.

68
Veena Pvt. Ltd., a small multiproduct company is taken over by a multinational company
( e.g. Hindustan Lever.) What changes in the control system would you expect and why?

Since Veena is a small multiproduct company it would require changes in control system which would
be related to transfer pricing a, as this company would generally provide inputs to HUL. Thus the domestic
operations generally involve transfer of goods and services only In view of this difference many other
considerations, in addition to the criteria used in domestic operations for the determination of transfer price, are
involved. These include:

(a) Fair Price: This is an important factor one needs to consider while determining the transfer price for foreign
operations. Companies that enter into joint ventures must ensure that the transfer price charged is fair. If such
companies charge a higher transfer price, it would reduce the profits of the joint venture and as a result reduce
the foreign partner's share of profits.

(b) Government Regulations: All countries have a regulatory framework under which business units operate.
Where government rules and regulations regarding transfer prices are lenient, the parent company should fix a
higher transfer price for all transfers to countries with high income tax rates. This approach would enable the
parent company to minimize taxes in such countries.

(c) Exchange Control Restrictions: Every country has foreign exchange control regulations.
These regulations impose a limit on the amount of foreign exchange available for the import of certain goods. To
accommodate the foreign subsidiary the parent company may have a lower transfer price so that the subsidiary is
able to import a larger quantity of required goods.

(d) Income Tax Regulations: The rates of income tax vary from country to country. To overcome this difference
the transfer price should be so fixed that countries with low tax rates show profits while others end up with a
loss. This helps the parent company to reduce its taxes on a global basis.

(e) Desire to accumulate funds: A company that wishes to accumulate funds in a particular country may fix the
transfer prices in such a manner that it facilitates shifting of funds into that country.

(f) Tariffs- and Duties: No country likes high imports. In order to restrict imports countries impose restrictions
such as quantitative restrictions, high duties and tariffs and banning import of products. The general practice is to
charge import duties as a percentage of the value of products imported, although a lower tariff may be levied if
the import value is lower. It is seen that the impact of tariffs on the profitability of foreign operations is generally
the reverse of the incidence of income taxes in transfer pricing. As such a low transfer price would lead to low
import duties on transfer, the profit arising in that country would be high. This results in high income taxes in
that country. It is therefore advisable that companies must compute the net effect of these factors while
determining transfer prices.

In designing performance evaluation systems for acquired Veena company,HUL could use the following
guidelines
Subsidiary managers should not be held responsible for translation effects. The simplest way to achieve this
objective is to compare budgets and actual results using the same metric and isolate inflation-related effects
through variance analysis. It is pointless for managers to worry about the appropriate metric. The MNE should
choose whatever metric is more convenient.

Transaction effects are best handled through centralized coordination of the MNE's overall hedging needs. This
is likely to be cheaper and simpler, and it prevents the subsidiary manager from becoming a foreign exchange
rate forecaster and speculator.

The subsidiary manager should be held responsible for the dependence effects of exchange rates resulting from
economic exposure.

Evaluation of the subsidiary as a basis for a decision to locate operations in a country or to relocate operations
from a country should reflect the consequences of translation. Transaction and economic exposures.

69
What is a Non - Profit Organization? How is the performance of this organization evaluated?

A nonprofit organization, as defined by law, is an organization that cannot distribute assets or income
to, or for the benefit of, its members, officers, or directors. The organization can, of course, compensate its
employees, including officers and members, for services rendered and for goods supplied. This definition does
not prohibit an organization from earning a profit; it prohibits only the distribution of profits. A nonprofit
organization needs to earn a modest profit, on average, to provide funds for working capital and for possible
“rainy days.”

Performance evaluation of nonprofit organization


For any organization, the most important reasons to measure performance are to improve effectiveness and to
acquire information that will allow the organization to drive its agenda forward. If the motivation for doing
evaluation remains outside an organization, the evaluation will have limited impact. To do performance
assessment effectively, an organization must commit to adopting a culture of measurement, because acceptance
must come from senior management, staff, funders, and board members alike.

➢ Board self-evaluation
Members of the Board of Directors should regularly evaluate the quality of their activities on a regular basis.
Activities might include staffing the Board with new members, developing the members into well-trained and
resourced members, discussing and debating topics to make wise decisions, and supervising the CEO. Probably
the biggest problem with Board self-evaluation is that it does not occur frequently enough. As a result, Board
members have no clear impression of how they are performing as members of a governing Board. Poor Board
operations, when undetected, can adversely affect the entire organization.

➢ Staff and volunteer (individual) performance evaluation


Most of us are familiar with employee performance appraisals, which evaluate the quality of an individual’s
performance in their position in the organization. Ideally, those appraisals reference the individual’s written job
description and performance goals to assess the quality of the individual’s progress toward achieving the desired
results described in those documents. Continued problems in individual performance often are the results of poor
strategic planning, program planning and staff development. If overall planning is not done effectively,
individuals can experience continued frustration, stress and low morale, resulting in their poor overall
performance. Experienced leaders have learned that continued problems in performance are not always the result
of a poor work ethic – the recurring problems may be the result of larger, more systemic problems in the
organizations.

➢ Program evaluation
Program evaluations have become much more common, particularly because donors demand them to ensure that
their investments are making a difference in their communities. Program evaluations are typically focused on the
quality of the program’s process, goals or outcomes. An ineffective program evaluation process often is the
result of poor program planning – programs should be designed so they can be evaluated. It can also be the result
of improper training about evaluation. Sometimes, leaders do not realize that they have the responsibility to
verify to the public that the nonprofit is indeed making a positive impact in the community. When program
evaluations are not performed well, or at all, there is little feedback to the strategic and program planning
activities. When strategic and program planning are done poorly, the entire organization is adversely effected.

➢ Evaluation of cross-functional processes


Cross-functional processes are those that span several systems, such as programs, functions and projects.
Common examples of major processes include information technology systems and quality management of
services. Because these cross-functional processes span so many areas of the organization, problems in these
processes can be the result of any type of ineffective planning, development and operating activities.

➢ Organizational evaluation

70
Ongoing evaluation of the entire organization is a major responsibility of all leaders in the organization. Leaders
sometimes do not recognize the ongoing activities of management to actually include organizational evaluations
– but they do. The activities of organizational evaluation occur every day. However, those evaluations usually
are not done systematically. As a result, useful evaluation information is not provided to the strategic and
program planning processes. Consequently, both processes can be ineffective because they do not focus on
improving the quality of operations in the workplace.

A Well Diversified company – Pritam International Ltd. sells one of its divisions to a group of its own
company managers. Explain what significant changes in systems and control procedures can be
expected? Why?

As, we Pritam International is a well diversified company. Sometimes, excessive diversification and that too in
unrelated lines of business causes failure in the business operations. One of the major reason for failures of many
Mergers and Diversification is excessive diversification. As, excessive diversification is ominous especially, in
unrelated lines of business. As, there may be no advantage of operating synergy. Neither through:

I) Sharing common resources nor


II) Sharing common core competencies

Therefore, it may be a strategic decision by the promoters and directors of the company to sell one of its
divisions. As, this may be impacting their core business. Sometimes, your core business tends to get neglected
mainily due to excesive diversification. As, the division is being sold to its own company managers. There,
might not be major changes in management control and systems. As, most of its managers will be the same. But,
they will have more autonomy to take decisions independently after acquisition. Now there will be less red
tapism and managers can take more risk. The managers will manage the firm in their own style. As, they are not
answerable to their superiors.

Currently, they are answerable to their stake-holders. As, the management is completely in their hands and that
too with full autonomy. The management might have identified the flaws in the previous controls and systems of
the company because of which the company might not be so effective and efficient. As, they have been
associated with the company over aperiod of time. They have a better understanding about the business
dynamics and environment in which the firm operates. So, they can take necessary steps to overcome the flaws
and improve the management control and systems.

So, that is why there will be some significant changes in the management control and systems and procedures if
there is further scope for improvement.

A TV dealership Veena Television (VT) is organized into four profit centers. colour TV, Black and
White, spare parts(SP) and servicing (SG) each headed by manager BTV in addition to BVTV sales;
also sells old TV exchanged (under scheme) by customer while purchasing new TV . in one particular
instance a new TV was sold for 14150(financed by cash rs2000, Bank loan 7350and Rs 4800;exchange
price for old TV agreed by CTV manager )cost of new TV was Rs 11420.Shivangi Manager of BTV,
examined the old TV (valued at Rs 3500 by TV trade magazine) and felt that she could get Rs 5000 for
that TV offer repairing cabinet, resulting and servicing for which she would use services of SP and SG
price chargeable to BTV by SP and SG are at market rates Rs235 for parts by SP and Rs 470 for
services by SG. Market price are arrived at after marking up cost by 3.5 times SG and 1.4 times SP.
BTV pays a service commission of Rs 250 per TV sold .overhead fixed per sale are CTV Rs 835;BTV Rs
665;SP RS 32 ;SG Rs 114.

Compute the profitability of the transaction assuming sales commission of $250 for the trade in on a selling price
of $5000

• Compute at market price


• At cost price
• Gross and net profit each

SOLUTION:
• SP of New TV by CTV = $14150.
• Original cost= $11420
71
• ($14150= $2000 cash down payment + $4800 trade in allowance + $7350 bank loan)
• Guide Book Value =$3500
• Ms. Shivangi of BTV Dept, believed that she could sell the trade in at $5000
• Other Cost: Rs235 for parts by SP and Rs 470 for services by SG

When trade-in is recorded @ $4800


4800+470+235=5505; 5000-5505= (-505)

Particulars New TV OLD TV Service Parts


Sales 14150 5000 470 235

Selling commission 0 250 0 0

Gross profit 2730 -505 470 235

Overhead 835 665 114 32

Servicing 0 470 0 0
Net profit before common exp 1895 -1640 591 123
If the trade-in is recorded @ $3500

Particulars New TV OLD TV Service Parts

Sales 14150 5000 470 235

Selling commission 0 250 0 0

Gross profit 2730 1045 470 235

Overhead 835 665 114 32

Servicing 0 470 0 0

Net profit before common exp 1895 -340 356 123

Why Balance Score Card is considered superior to other methods of Performance Appraisal?
Prepare Balance Score Card for any organization you are familiar with.
What is the Balanced Scorecard?

The rationale for the development of the Balanced Scorecard was a growing dissatisfaction with
traditional, financial measures of performance. These measures suffer from a number of serious drawbacks in
72
that they take a short-term, lagged (i.e., historic) view of performance. The shift towards flexible, lean
production/service systems in many firms has strengthened the requirement for performance measurement
systems to become more broadly based, incorporating both non-financial and external measures of performance.
According to Kaplan and Norton, the Balanced Scorecard provides a better assessment of performance as it
"enables companies to track financial results while simultaneously monitoring progress in building the
capabilities and acquiring the intangible assets they need for future growth".
The original scorecard designed by Kaplan and Norton contained four key groupings of performance measures.
These four groupings, called ‘perspectives’ by Kaplan and Norton, were considered sufficient to track the key
drivers of both current and future financial performance of the firm. The perspectives focused on the
achievements of the firm in four areas: namely the financial, customer, internal business process and
innovation/learning perspectives. The four perspectives can be represented as an interlinked hierarchy. The
firm’s strategy underlies the whole scorecard, as the measures for each of the four perspectives are drawn from
this strategy.

To obtain a satisfactory overview of performance, the scorecard will require a mix of lagging and leading
(forward looking) measures. Financial measures tend to be lagged and consequently, the measures chosen for the
other perspectives will need to include leading measures. In general, outcome measures tend to be lagged, for
example, current market share is the result of past decisions and consequently is a lagging measure. Thus the
challenge in designing a Balanced Scorecard is to choose driver measures which lead changes in the outcome
measures in the non-financial perspectives and which ultimately drive the financial measures.
Once the firm’s objectives have been agreed and the appropriate outcome and driver measures chosen for each of
the perspectives, firm and managerial performance is assessed by comparing actual attainment on each measure
with the target set for that measure.
Objective Measure Target Actual

Benefits from adopting the Balanced Scorecard

There are several benefits from implementing a Balanced Scorecard. Originally the Balanced Scorecard
was seen as a useful tool for performance measurement. In this role, the Balanced Scorecard was seen as
integrating financial/non-financial, internal/external and leading /lagging information on firm performance in a
coherent fashion.
Later it was realised that the Balanced Scorecard could play a pivotal role in the strategic management
process. Because the Balanced Scorecard requires management to clarify and obtain consensus on the strategic
objectives of the firm, it can assist in the communication of the chosen strategy, consequently aligning the efforts
both of individuals and of departments. In this role, there is a clear link between the Balanced Scorecard and
management by objectives (MBO). Effective implementation of a Balanced Scorecard project will generally
involve the development of a series of hierarchical (cascaded) scorecards. Given the overall corporate scorecard,
supporting scorecards can be developed for each department within the firm. Within each department, a
scorecard can be developed for each manager (or perhaps even for each individual member of staff) which links
the objectives on each perspective for that manager back to the objectives for each perspective outlined in the
scorecard for the department and finally, back to the objectives listed in the firm’s overall scorecard.

73
The Balanced Scorecard could be used to assist in corporate restructuring. In recent years, many firms have
migrated away from a traditional hierarchical structure to a flatter, team-based organisational structure. The
Balanced Scorecard can support such changes, as it can help clarify the objectives and the critical success factors
for the newly formed teams.
Apart from the communication and co-ordination roles of the Balanced Scorecard in strategic implementation,
the Balanced Scorecard can be used to link strategy to specific critical success factors in the customer,
internal business process and growth/learning perspectives. By setting both short and long-term targets for
driver and outcome measures and by comparing actual attainment against target, feedback is obtained on how
well the strategy is being implemented and on whether the strategy is working.
Building on the Balanced Scorecard’s use as a strategic management tool, it has been suggested that the
Balanced Scorecard can play a role in the investment appraisal process.
Traditional methods of investment appraisal such as discounted cash flow do not cope well with
investments which generate indirect rather than direct financial returns. Examples of these include investments
which enhance the future ‘flexibility’ of a firm or investments in the firm’s infrastructure, such as an enhanced
management information system. The Balanced Scorecard can assist management’s investment appraisal
decisions as it provides managers with a mechanism to incorporate the strategic aspects of the investment into
the appraisal process. This could be achieved by using a weighting system developed from a firm’s Balanced
Scorecard measures to evaluate new projects. An index score would be calculated for each investment
opportunity and projects would then be ranked and selected based on this score.
Balance Score Card of Credit Card Company

Soniya Company has two Divisions: A & B. Return on Investment for both divisions is 20%.
Analyse and comment on divisional performance of each.

Details are given below.

Particulars Div A Div B


Divisional sales 4000000 9600000
Divisional Investment 2000000 3200000
Profit 400000 640000

SOLUTION
74
As Profit Margin = Profit *100
Sales

Profit Margin for Division ‘A’= 4,00,000 /40,00,000 *100 = 10%

Profit Margin for Division ‘B’ = 6,40,000/ 96,00,000 *100 = 6.6%

Turnover of Investment = Sales * 100


Investment

Turnover of Investment for Division ‘A’ = 40,00,000/20,00,000 = 2 times

Turnover of Investment for Division ‘B’ = 96,00,000/32,00,000 = 3 times

As Return on investment for both Divisions A and B is 20%.

COMMENTS:-

Division ‘A’ – Although ‘A’ has more profit margin than Division ‘B’ that is 10% as compared to 6.6% of ‘B’,
so it has more profitability but inspite of it, division ‘A’ has lower turnover of investment that its assets
management is bad than Division ‘B’, it can be improved by increased sales or reducing investment.

Division ‘B’ – Needs to improve profit margin by increasing sales and reduce variable cost and sales at same
price or by reducing salesprice and increase the volume of sales so that its profit would improve. As it has good
assets management shown by its turnoverof Division ‘B’ that is 3 times which is better than Division ‘A’. So it
can become profitable organisation by improving Profit Margin.

Two divisions A and B of sonali enterprises operate Profit centers. Div A normally purchases annually
10000 nos. of required components from Div B, which has recently informed Div A that it will increase
selling price p.u to Rs. 1100. Div A decided to purchase the components from open market available at
Rs.1000 p.u Div B is not happy and justified its decision to increase price due to inflation and added that
the overall company profitability will reduce and decision will lead to excess capacity in Div B, whose
V.C and Fixed cost p.u. are Rs. 950 and Rs.1100.
1. Assuming that no alternate use exists for excess capacity in Div B, will company benefit as a
whole if Div A buys from the market.
2. If the market price reduces by Rs.80 p.u. What would be the effect on the company (assuming
Div B has still excess capacity) if A buys from market.
3. If excess capacity of Div B could be use for alternative sales at yearly costs savings of Rs. 14.5
lacs, should Div A purchase from outside?
Justify your answers with figures.

SOLUTION

1) Division ‘A’ action

BUY OUTSIDE (Rs.) (Rs.) BUY INSIDE

Total Purchase Cost 10,00,000 Nil


Total Outlay Cost Nil 9,50,000

Net Cash Outflow To 10,00,000 9,50,000


The Company As A
75
Whole

The Company as a whole will benefit if Division ‘A’ buys inside from Division ‘B’.

2) If the market price reduces by Rs.80 p.u


Division ‘A’ action
BUY OUTSIDE (Rs.) (Rs.) BUY INSIDE

Total Purchase Cost 9,20,000 Nil


Total Outlay Cost Nil 9,50,000

Net Cash Outflow To 9,20,000 9,50,000


The Company As A
Whole

The Company as a whole benefit if ‘A’ buys from outside supplier at Rs. (1000-80) = 920

3) If excess capacity of Div B could be use for alternative sales at yearly costs savings of Rs. 14.5 lakhs

Division ‘A’ action


BUY OUTSIDE (Rs.) (Rs.) BUY INSIDE

Total Purchase Cost 10,00,000 Nil

Total Outlay Cost Nil 9,50,000

Revenue From Using 1,45,000


These Facilities
Net Cash Outflow To 8,55,000 9,50,000
The Company As A
Whole
Therefore, Company should purchase from outside.
Discuss and illustrate differences and similarities between

Strategy Formulation and Management Control

Characteristics Strategy Formulation Management Control

a) Focus of plan On one aspect at a time On entire organisation

b) Complexities Many variables hence complex Less complex

c) Nature of information Tailor-made for the issue, more Integrated, more internal and
external and predictive, less historical, more accurate.
accurate.

d) Structure Unstructured and irregular, each Rhythmic, definite pattern, set


problem being different procedure

e) Communication of information Relatively simple Relatively difficult

f) Purpose of estimates Show expected results Lead to desired result

76
g) Persons involved Staff and top management Line and top management

h) No. of persons involved Small Large

i) Mental activity Creative, analytical Administrative, persuasive

j) Planning and control Planning dominant but some Emphasis on both planning and
control control

k) Time horizon Tends to be long Tends to be short

l) End result Policies and precedents Action within policies laid

m) Appraisal of job done Extremely difficult Less difficult

Management Control and Task Control

Characteristics Task Control Management control


a) Focus of plan Single task or transaction On entire organisation
b) Nature of information Tailor-made to operation, specific, Integrated, more internal and
often non- financial, real time historical, more accurate
c) Persons involved Supervisors Line and top management
d) Mental activity Follow directives or none as in Administrative, persuasive
case of machines or set objectives
e) Time horizon Day to day Tends to be short
f) Type of cost Engineered- Existence of objective Discretionary- Control is more
standard against which actuals can difficult due to subjective
be compared makes control easier. consideration.

Girish Engineering Ltd. (Numerical) (MCS-2006)

(1) On the basis of costing, will the manager be interested in accepting the market offer?

Solution:

Particulars Amount (Rs./unit) Amount (Rs./unit)

Cost of critical component for 220


division X

Cost of other material 500

77
Fixed & processing costs 290

Total cost for division X 1010

Selling price of final product 1000

Net loss for division X 10

Desired profit for division X 60

Thus on the basis of full actual cost incurred by division X, it would suffer a loss of Rs.10/unit if it accepts the
market offer whereas its target profit margin is Rs.60/unit. So, division X would not accept the market offer.

(2) Is this offer beneficial to the company as a whole? Justify with figures.

Solution:

Particulars Amount (Rs. Lakh) Amount (Rs. Lakh)

Cash inflow (a) 50 (5000 units * Rs.1000/unit)

Cash outlay:

Variable cost for division Y 5 (Working note)

Material bought by division X from 25 (5000 units * Rs.500/unit)


outside

Total cash outlay (b) 30

Net cash inflow to Company as a 20


whole [(a)- (b)]

Thus, the Company as an entity would receive cash inflow of Rs.20 lakh. So, the offer is beneficial to the
company as a whole.

Working notes:-

Variable cost for division Y:

Desired RoI =10% of Rs.2.4 Cr. p.a. = Rs.24 lakh p.a. i.e. Rs.2 lakh per month

Fixed cost assigned to division X = Rs.4 lakh per month

Fixed cost p.u. = 400000/5000 = Rs.80

Contribution per month = Rs.6 lakh

Total sales value for division Y = 220 * 5000 = Rs.11 lakh per month

So, total Variable cost per month for division Y = 11 lakh – 6 lakh = Rs.5 lakh

Variable cost p.u. for division Y = 500000/5000 = Rs.100

An annual investment of Rs2.4 Cr. is assigned by division Y to division X but it does not imply that a
special investment of Rs.2.4 Cr. is made by division Y exclusively to produce the component required by

78
division X. Therefore, cash outflow associated with this investment is not relevant for the above concerned
decision regarding accept the market offer.

(1) If yes, how should the company organize its transfer pricing mechanism? Illustrate.

Solution:

Currently, Girish Engineering Ltd. is following 2 step transfer pricing method wherein the selling division
charges actual variable cost along with profit mark-up & separately allocates a particular amount of fixed
costs per month to the buying division. However, in the case of division X (buying division) & division Y
(selling division), this method of transfer pricing is not feasible as division X would suffer loss if it accepts
the market offer under this scenario. So, divisions X & Y can negotiate a transfer price by taking into
account full actual variable cost (Rs.100 p.u.) & half of fixed costs incurred by division Y that is assigned to
division X (Rs.40 p.u.) & add a mark-up of say Rs.10/unit. Taking into consideration only half of the fixed
costs of selling division i.e. division Y prevents shifting of any operational inefficiencies from selling
division to buying division i.e. division X, which would unnecessarily increase the costs for division X and
thereby eat up its profit margin. In this case, division X’s total costs would turn out to Rs.940 (500 + 290 +
150) & would earn a profit margin of Rs.60 p.u. (desired profit margin). Also, contribution p.u. for division
Y would be Rs.50 (150 – 100). Thus, total contribution for division Y would be Rs.250000 resulting in RoI
of 12.5% (250000/2000000) which is more than the desired RoI of 10%.

Suresh Ltd. (Numerical) (MCS-2007)

(a) Define profit in this case and prepare a statement for both divisions and overall company.

Solution:

i) Profitability statement of Division A:-

Particulars Amount(Rs.)
Selling price p.u. 35
Variable Cost p.u. 11
Contribution p.u. 24

Contribution p.u. Expected sales (no. Total contribution Total Fixed cost Net profit (Rs.)
of units) (Rs.)
24 2000 48000 60000 (12000)
24 3000 72000 60000 12000
24 6000 144000 60000 84000

ii) Profitability statement of Division B:-

Selling p.u. Total Contribution Expected Total Total Fixed Net profit
variable cost p.u. sales (no. of contribution cost (Rs.) (Rs.)
p.u. units)
90 42 48 2000 96000 90000 6000
80 42 38 3000 114000 90000 24000
50 42 8 6000 48000 90000 (42000)
[Note: Total Variable cost p.u. = Variable cost p.u. (Rs.7) + Transfer price of intermediate product (Rs.35)]

iii) Profitability statement of Company as a whole:-

Expected sales Net profit of division A Net profit of Division B Total Net profit
(Rs.) (Rs.)
79
2000 (12000) 6000 (6000)
3000 12000 24000 36000
6000 84000 (42000) 42000

(b) State the selling price which maximizes profits for division B and company as a whole. Comment on
why the latter price is unlikely to be selected by division B.

Solution:

As per the calculation in part (a), selling price p.u. of Rs.80 maximizes profit for division B whereas selling
price p.u. of Rs.50 maximizes profit for the Company as a whole. However, if Division B opts for selling
price p.u. of Rs.50 in order to maximize Company’s profit, it would suffer a loss of Rs.42000. Therefore,
Division B would not select Selling price p.u. of Rs.50.

Explain different organizational goals. Comment on goal of shareholder value maximization in


particular.

Goals

Although we often refer to the goals of a corporation, a corporation does not have goals; it is an artificial being
with no mind or decision-making ability of its own. Corporate goals are determined by the chief executive
officer (CEO) of the corporation, with the advice of other members of senior management, and they are usually
ratified by the board of directors. In many corporations, the goals originally set by the founder persist for
generations. Examples are Henry Ford, Ford Motor Company; Alfred P. Sloan, General Motors Corporation;
Walt Disney, Walt Disney Company; George Eastman, Eastman Kodak; and Sam Walton, Wal-Mart.

Economic Goals

Shareholder's value, Earning per share and Market value, all relate to maximizing shareholder's value, which is
not a desirable goal, because what is 'maximum' is difficult to determine. Although optimizing shareholder value
may be one goal, but there are other stakeholders in the business also such as customers, employees, creditors,
community and so on. Again, shareholder value is usually equated with the market value of the company's stock.
But market value is not an accurate measure of the worth of shareholders' investments. Besides, such value can
be obtained only when the share is traded in the stock exchange.

It is interesting to note that Henry Ford's operating philosophy was 'satisfactory profit', not 'maximum
profit'. He said, "A reasonable profit is right, but not too much. So, it has been my policy to force the price of the
car down as fast as production would permit and give the benefit to the user and laborers, with resulting
surprisingly enormous benefit to ourselves"

Other goals such as adding new products, or product-line or new business actually indicate normal
organizational growth.

Social Goals

However, every organization has its share of responsibility towards the local community where it is
situated, and the public at large. It is very difficult to incorporate in Management Control System such goals as
taking pride in an organization which cares for the society and renders service to the public. Of course, any
concrete structural programme indicating its operational expenses, methods of providing service, personnel
involved in rendering service and the nature of the service in details can, however, be mentioned through an
appropriate system.

Profitability

In a business, profitability is usually the most important goal.

80
Return on investment can be found by simply dividing profit (i.e., revenues minus expenses) by investment,
but this method does not draw attention to the two principal components: profit margin and investment
turnover.
In the basic form of this equation, "investment" refers to the shareholders' investment, which consists of
proceeds from the issuance of stock, plus retained earnings.

One of management's responsibilities is to arrive at the right balance between the two main sources of
financing: debt and equity. The shareholders' investment (i.e., equity) is the amount of financing that was not
obtained by debt, that is, by borrowing. For many purposes, the source of financing is not relevant;
"investment" thus means the total of debt capital and equity capital.
"Profitability" refers to profits in the long run, rather than in the current quarter or year. Many current
expenditure (e.g., amounts spent on advertising or research and development) reduce current profits but increase
profits over time.
Some CEOs stress only part of the profitability equation. Jack Welch, former CEO of General Electric
Company, explicitly focused on revenue; he stated that General Electric should not be in any business in which
its sales revenues were not the largest or the second largest of any company in that business. This does not imply
that Welch neglected the other components
of the equation; rather, it suggests that in his mind there was a close correlation between market share and
return on investment.

Other CEOs, however, emphasize revenues for a different reason: For them, company size is a goal. Such a
priority can lead to problems. If expenses are too high, the profit margin will not give shareholders a good
return on their investment. Even if the profit margin is satisfactory, the organization may still not earn a good
return if the investment is too large.

Some CEOs focus on profit either as a monetary amount or as a percentage of revenue. This focus does not
recognize the simple fact that if additional profits are obtained by a greater than proportional increase in
investment, each dollar of investment has earned less.

Maximizing Shareholder Value

In the 1980s and 1990s the term shareholder value appeared frequently in the business literature. This concept is
that the appropriate goal of a for-profit corporation is to maximize shareholder value. Although the meaning of
this term was not always clear, it probably refers to the market price of the corporation's stock. We believe,
however, that achieving satisfactory profit is a better way of stating a corporation's goal, for two reasons.

First, "maximizing" implies that there is a way of finding the maximum amount that a company can earn. This
is not the case. In deciding between two courses of action, management usually selects the one it believes will
increase profitability the most. But management rarely, if ever, identifies all the possible alternatives and their
respective effects on profitability. Furthermore, profit maximization requires that marginal costs and a demand
curve be calculated, and managers usually do not know what these are. If maximization were the goal, managers
would spend every working hour (and many sleepless nights) thinking about endless alternatives for increasing
profitability; life is generally considered to be too short to warrant such an effort.

Second, although optimizing shareholder value may be a major goal, it is by no means the only goal for most
organizations. Certainly a business that does not earn a profit at least equal to its cost of capital is not doing its
job; unless it does so, it cannot discharge any other responsibilities. But economic performance is not the sole
responsibility of a business, nor is shareholder value. Most managers want to behave ethically, and most feel an
obligation to other stakeholders in the organization in addition to shareholders.
Example: Henry Ford's operating philosophy was satisfactory profit, not maximum profit. He wrote let me
say right here that I do not believe that we should make such an awful profit on our cars. A reasonable
profit is right, but not too much. So it has been my policy to force the price of the car down as fast as
production would permit, and give the benefits to the users and laborers-with resulting surprisingly
enormous benefits to ourselves.

By rejecting the maximization concept, we do not mean to question the validity of certain obvious principles.
A course of action that decreases expenses without affecting another element, such as market share, is sound. So
is a course of action that increases expenses with a greater than proportional increase in revenues, such as
expanding the advertising budget. So, too, are actions that increase profit with a less than proportional increase
in shareholder investment (or, of course, with no such increase at all), such as purchasing a cost-saving machine.

81
These principles assume, in all cases, that the course of action is ethical and consistent with the corporation's
other goals.
An organization's pursuit of profitability is affected by management's willingness to take risks. The degree of
risk-taking varies with the personalities of individual managers. Nevertheless there is always an upper limit;
some organizations explicitly state that management's primary responsibility is to preserve the company's assets,
with profitability considered a secondary goal. The Asian .financial crisis during 1996-1998 is traceable, in large
part, to the fact that banks in Asia's emerging markets made what appeared to be highly profitable loans without
paying adequate attention to the level of risk involved.

Multiple Stakeholder Approach

Organizations participate in three markets: the capital market, the product market, and the factor market. A firm
raises funds in the capital market, and the public stockholders are therefore an important constituency. The firm
sells its goods and services in the product market, and customers form a key constituency. It competes for
resources such as human capital and raw materials in the factor market and the prime constituencies are the
company's employees and suppliers and the various communities in which the resources and the company's
operations are located.

The firm has a responsibility to all these multiple stakeholders-shareholders, customers, employees, suppliers,
and communities. Ideally, its management control system should identify the goals for each of these groups and
develop scorecards to track performance.
Example: In 2005, the Acer Group, headquartered in Taiwan, was one of the largest computer companies
The Company subscribed to the multiple stakeholder approach and managed its internal operations to
satisfy the needs of several constituencies. To quote Stan 'Shih,-the founder, "The customer is number 1,
the employee is number 2, the shareholder is number 3. I keep this message consistent with all my
colleagues. I even consider the company's banks, suppliers, and others we do business with are our
stakeholders; even society is stakeholder. I do my best to run the company that way."

Lincoln Electric Company is well known for its philosophy that employee satisfaction was more important
than shareholder value. James Lincoln wrote: "The last group to be considered is the stockholders who own
stock because they think it will be more profitable than investing more in any other way. The absentee
stockholder is not' of any value to the customer or to the worker, since he has no knowledge of nor interest in the
company other than greater dividends and advance in the price of his stock." Donald F. Hastings, chairman and
chief executive officer, emphasized that this was still the company's philosophy in 1996.

Explain and illustrate with one example differences between 3 forms of internal audit
Financial, Operational & Management.

Financial Audit

Financial Audit is a historically oriented, independent evaluation performed by internal auditor or external
auditor for the purpose of attesting to the fairness, accuracy and reliability of the financial data, providing
protection for the entity's assets; evaluating the adequacy and accomplishment of the system (internal control)
designed, provide for the aforementioned Fairness and Protection, Financial data, while not being the only
source of evidence, are the primary evidential source. The evaluation is performed on a planned basis rather
than a request".

Institute of Internal Auditor:-


Financial audit takes care of the protective aspect of the business and it does not normally carry out
constructive appraisal function of the business operations. It helps in detection and prevention of fraud. It also
verifies whether documentation and flow of activities arc in conformity with the internal control system
introduced and developed within the organization. It helps coordinating with statutory auditor to help them in
proper discharge of their function. Besides, financial audit also ensures compliance with statutory laws
especially in financial and accounting matters.

82
Objectives of Financial Audit:

• To see that established accounting systems and procedures have been complied with
• To see that proper records have been maintained for the fixed assets of the Concern to look into correctness
of the financial data and records along with correctness of the accounting procedure followed.
• To see whether scrap, salvage and surplus materials have been properly accounted for etc.
• To see that internal control system has been working properly.
• To see that any abrupt variation in sales, purchases etc.; with respect to immediate previous year are not
due to any irregularity
• To see that the credit control has been strictly followed.
• To see that all payments have been made with proper authorization and approval. .
• To see that preparation of salary and wage pay roll has been properly done.

The opinion expressed by the auditors shall be based on verified data, reference to ich shall also be made here
and, if practicable, included after the company has been
forded on opportunity to comment on them.

Management Audit

It is a complex task closely related with the process of management. It is highly result oriented. It requires
inter/multi-disciplinary approach as it involves examination, review and appraisal of various policies and actions
of management on the basis of certain norms/standards.
It undertakes comprehensive and critical review of all organizational activities with wider perspective.
It goes beyond conventional audit and audits the efficacy of the management itself.

Definition:
It's a comprehensive and constructive examination of an organization, the structure of a company, institution
or branch of government or of any components thereof, such as division or department and its plans, objectives,
its means of operations and its use of human and physical facilities. .

William P. Leonard

It's an investigation of a business from the higher level downwards in order to ascertain whether sound
management prevails throughout, thus facilitating the most effective relationship with the outside world and
the most efficient and smooth running internally.

Leslie Howard

It is an audit performed with the object of examining the efficacy of the institution/control systems,
management procedures towards the achievement of enterprise goals.

Churchill & Cyert

It is an objective and independent appraisal of the effectiveness of managers and the effectiveness of the
corporate structure in the achievement of company objectives and policies. Its aim is to identify existing and
potential management weaknesses within an organization and to recommend ways to rectify these
weaknesses.

Chartered Institute of Management Accountants London

Thus it can be seen that management audit is an examination, review and appraisal of the various policies
and actions of the management. It is a tool for the evaluation of methods and performance in all the areas of
the enterprise.

Objectives

83
• To ascertain the provision of proper control at different levels, their effectiveness I in accomplishing
management goals.
• Ascertain objectives of the organization are properly communicated and understood at all levels.
• To reveal defects or irregularities in any of the elements examined and to indicate what improvements are
possible to obtain the best results of the operations of the company.
• To assist the management to achieve the most efficient administration of its operations.
• To suggest to the management the ways and means to achieve the objectives if the management of the
organization itself lacks the knowledge of efficient management.
• It aims to achieve the efficiency of management and assess the strength and weaknesses of the organization
structure, its management team and its corporate culture.
• To ascertain the provision of proper control at different levels, their effectiveness in accomplishing
management goals.
• Ascertain objectives of the organization are properly communicated and understood at all levels.
• To reveal defects or irregularities in any of the elements examined and to indicate what improvements are
possible to obtain the best results of the operations of the company.
• To assist the management to achieve the most efficient administration of its operations.
• To suggest to the management the ways and means to achieve the objectives if the
• management of the organization itself lacks the knowledge of efficient management.
• It aims to achieve the efficiency of management and assess the strength and weaknesses of the organization
structure, its management team and its corporate culture.
• To help the management at all levels in the effective and efficient discharge of their duties and
responsibilities.

The auditor must apprise managerial performance at all levels of the organization. The audit starts right at
the top level of the management. It studies the managerial performance at all the levels of management. The
audit has to study the decision-making system of the organization and also the level of autonomy granted to the
managers at different levels of the organization. The authority and responsibility given at the different levels of
the management. One of the most important things that the audit must study is that the mangers at various levels
use the authority.

Conducting Management Audit

Management audit requires an interdisciplinary approach since it involves a review of all aspects of
management functions. It has to be conducted by a team of experts because this requires 3 varieties of skills,
which one individual may not possess.
The team may consist of management experts, accountants, and the operation research specialists, the
industry experts and even social scientists.
The auditors must have analytical mind and ability to look at a management function form the point of view
of the organization as a whole. They therefore have to be properly trained in this aspect. They need to have
through knowledge of the management science and they should be acquainted with the salient features of various
functional areas.

Under financial audit, the entire emphasis is on macro-aspect, the individual transactions being- scrutinized
for check of the aggregates. It is concerned with examination of transactions recorded in the books of account. It
reviews the procedure and internal checks, and scrutinizes individual transactions for the purpose of verification,
of Profit and Loss Account and Balance Sheet. Financial audit is not concerned with ~ avoidance of profiteering
motive. It indicates the financial position and over~ performance of the business, regardless of its performance in
various segments. Financial audit is applicable to all classes of companies and industries irrespective of size and
Dan of operations.
Instead of serving the interest of the management and the Government, it serves interest of shareholders.
Financial audit is organization - oriented. It is conducted under Sections 224 - 232 of the Companies Act 1956.

Financial Audit Management Audit


It is concerned with financial aspects of It is concerned with the review of the past
Performance to ascertain whether it is in tune with the
business transactions of the year under objectives, policies and procedures of the enterprise.
audit

84
The auditor examines the past financial The management auditor reports on performance of the
records to report his opinion on the truth management during a particular period and suggest ways
and fairness of the representations made in the financial to remedy the deficiencies, including modification of
statements. Examination of the performance of the objectives, policies etc.
management is
beyond his scope

Past year '(Financial) transactions are No limit as to the period to be covered


Covered Enterprises such as companies, trust and societies
etc.

There is legal compulsion as regards management audit.


Financial audit is compulsory in the case of certain
enterprises such as companies, trust and societies etc.
The auditor reports to the owner, i.e. The auditor reports to the management
shareholders in the Case of a company

Explain briefly various stages of management control process citing salient features of each.

Management control process involves communication of information to the managers at various levels
of hierarchy and their interactions arising out of them. These communications aim towards attaining the
organization's goals. But individual managers have their personal goals also. For example, a young manager with
good education, experience, personality and social background joins a company like Britannia Industries or
Reliance. The company finds him fit for the position as per job specifications, appoints him and makes him
aware of what the company expects of him. The young manager sets his goals of gaining rich experience for his
career progress besides adequate compensation packages. Naturally, his actions will be directed towards
achieving his own objectives and goals while serving the company. Thus, his self-interest and the best interest of
the organization are apparently in conflict. But the best results can be achieved by perfectly matching the two
interests and this is called 'goal congruence'.

It is quite apparent that perfect congruence between the goals of the individual and the organization individual's
goals and the organization's goals can never happen. Yet, the main purpose of a management control system is to
assure goal congruence between the interest of the individual and the organization as far as practicable.

Formal and Informal Communication

As mentioned earlier, all the communication of information may be either formal or informal. The formal
communication system involves strategic plan, budgets, standards and reports whereas the informal
communication is made through letters and memos, verbally or even by facial expression.
Formal communications are all documented and addressed to the responsible managers for their information and
actions, if necessary. However, the actions depend on the perception of the individual managers.

85
Informal communication, on the other hand, relates to some external factors-work ethics, management style and
culture. Added to these factors is the existence of an informal organization within the structured formal
organization.
Informality refers to the relaxation of sharp differentiation and explicit description of behavior as indicated
in the hierarchy and thereby, moving away from superior/subordinate relationship. However, such relations
depend on the personal capabilities of the manager such as education, experience, expertise, trust and
cooperation. For example, Accounts Manager of Nasik Plant (see the organization chart in the diagram 3.2)
reports to the General Manager of the Plant. While visiting the Corporate Office for attending a Training Course,
he meets other colleagues, parallel officers and even the Finance Director. The latter communicates some
important matter to him verbally and wants action thereon. Accounts Manager carried out the instructions so
given. As per the organization chart, he should inform his General Manager, but it depends on his own
perception of the situation, and he mayor may not report to the General Manager.

Work Ethics, Management Style and Culture

External factors like work ethics vary from place to place. Therefore, organization work culture depends on
the general behavior of the people in the society where the organization situates. Work culture generally differs
because of the life style and the attitude towards the work. For example, people of Mumbai lead very fast life.
Time has more value at Mumbai as compared to Kolkata, where people take things easily and leisurely. Japanese
and Korean people have reputation for their excellent work culture.

However, the most important internal factor is the organization's culture and climate. The culture refers to
the set of common beliefs, attitudes, norms, relationships and assumptions that are explicitly or implicitly
accepted and evidenced throughout the organization. The writer joined Union Carbide as an Assistant just three
days before Christmas Eve. On the very second day, when he attended Christmas lunch, his table was shared by
none other than the General Sales Manager Dr. W.R. Correa. He kept us amused with various stories of his
recent tour abroad and recited Urdu 'shairies', even sharing jokes. Such a situation was unthinkable in Jessop &
Co., where sharp differences were maintained at every level of hierarchy.

Management control systems

Climate is used to designate the quality of the internal environment that conditions the quality of
cooperation, the development of individuals, the extent of members' dedication or commitment to organizational
purpose and the efficiency with which that purpose is translated into results. Climate is the atmosphere in which
individuals work help, judge, and reward, constrain and find out about each other. It influences moral-the
attitude of the individual towards his/her work and environment.

Culture differs between the organizations, but cultural norms are extremely important. They are not written
like formal communication. But the existence of a good culture can be felt from the behavior of the members of
the organization. Once the writer landed up with his family at Hyderabad in the early morning to discover that
nobody had come to receive them at the station. His visit was arranged through non other than the Director of the
company himself. His unit being new, telephone directory did not include any number of his unit, but the parent
organization's telephone number was located. When an executive of the parent company was contacted, he
immediately sent an officer of the company with a car to pick us up to their Guest House, entertain with coffee
and then put up in a Hotel. What subsequently happened is a different matter, but the attitude and treatment of
that member of organization speak volumes about their excellent culture.

In any organization, the culture remains unchanged as long as the Chief Executive remains in position.
When a new executive replaces him, there is likelihood of some change in the culture, unless the new Chief
follows the footsteps of his predecessor and maintains it. Generally, if higher positions are filled in through
promotion of internal executives, the culture remains unchanged and the traditions are maintained.

The other important internal factor which influences management control system is management style-that
is the attitude of the superior to his subordinates and the latter's reaction through their perception of the attitude
of their superiors. Again, the attitude ultimately stems from the temperament of the Chief Executive, who
controls the entire organization. That is why R. W. Emerson said "an institute is the lengthened shadow of a
man".

86
Importance of Informal Communication

An organization indulges in informal control process when encountering non-routine decision-making or when
seeking new information to increase understanding of some problem areas. During a very critical period in an
organization, the writer found that the Chief Executive used to call managers informally at his residence or club
to extract information in a relaxed manner rather than in a tense situation prevailing in the factory.

Formal Control Process

Formal communication system is structured as per the 'hierarchy outlined in the organization chart. The system
has the following four components:

(a) Strategic Plan and Programme

The foundation of management control process lies in the organization's goals and its strategies for
attaining these goals. A strategic plan is prepared in order to implement the strategies, after carefully considering
opportunities and threats in the external environment as well as the strengths and weaknesses in the internal
environment. Thus, a strategic plan and programme is prepared as a guideline to budgeting.

(b) Budgeting

The strategic plan is converted to an annual budget incorporating planned expenditure and revenues for
individual responsibility centers. Expenses and revenues are marked for each responsibility centre period wise,
say monthly, quarterly, half yearly, and annually.

(c) Operations and Measurement

Responsibility centers operate within the framework of the budget, established standards, standing instructions,
practices and operating procedures embodied in 'rules', and 'manuals'. Thus, besides budget, the responsibility
centers are also guided by a large number of rules. They record the resources actually used and revenue earned.
They also classify the data by programmes as well as by responsibility centers for performance measurement.

(d) Reporting

Actual performance is analyzed, measured and reported against plan, indicating variances and highlighting areas
of weaknesses. If the performance is satisfactory, feedback information is sent to the responsibility centre
concerned for praise or reward. If the same is unsatisfactory feedback communication is sent to the responsibility
centre concerned for corrective action. If such action requires to be included in the budget, then the latter is
revised to give effect to the changed position. If required, then the plan itself can be revised and a new basis of
control may be established.
The aforesaid formal control process has been presented in the following diagram:

87
88

Vous aimerez peut-être aussi